Test 2 NUR 160

Réussis tes devoirs et examens dès maintenant avec Quizwiz!

he nurse is assessing a client at a postpartum visit and notes the client is emotionally sensitive, complains about being a failure, and appears extremely sad. The nurse concludes the client is presenting with which potential condition? 1. anxiety disorders 2. postpartum blues 3. postpartum psychosis 4. postpartum depression

4. postpartum depression

The nurse is performing an assessment on a 2-day postpartum client and discovers a boggy fundus at the umbilicus and slightly to the right. The nurse determines that this is most likely related to which situation? 1. Bladder distention 2. Poor bladder tone 3. Uterine atony 4. Full bowel

1. Bladder distention

The nurse can expect a client who had a cesarean birth to have less lochia discharge than the client who had a vaginal birth. 1. True 2. False

1. True Women who had a cesarean birth will have less lochia discharge than those who had a vaginal birth, but stages and color changes remain the same.

A nurse is assessing a newborn for self-regulation. Which of the following would the nurse interpret as indicating self-regulation? 1. infant moves hand to mouth 2. infant is crying vigorously 3. infant is making kicking motions 4. infant is lying quietly

1. infant moves hand to mouth Self-regulation is a form of self-soothing for an infant (e.g., sucking on hands, using hand-to-mouth type movements).

Which maternal factors should the nurse consider contributory to a newborn being large for gestational age? Select all that apply. 1. postdates gestation 2. prepregnancy obesity 3. diabetes 4. alcohol use 5. renal infection

1. postdates gestation 2. prepregnancy obesity 3. diabetes Diabetes, postdates gestation, and prepregnancy obesity are the maternal factors the nurse should consider that could lead to a newborn being large for gestational age. Renal condition and maternal alcohol use are not factors associated with a newborn being large for gestational age.

Blood work results show that a client has a luteal defect, which is suspected to be the primary cause of her subfertility. The nurse explains that this condition is generally corrected by which of the following? 1. progesterone vaginal suppositories 2. myomectomy 3. diathermy 4. clomiphene citrate

1. progesterone vaginal suppositories If the problem of subfertility appears to be a luteal phase defect, this can be corrected by progesterone vaginal suppositories begun on the third day of a woman's temperature rise and continued for the next 6 weeks (if pregnancy occurs) or until a menstrual flow begins. If a myoma (fibroid tumor) or intrauterine adhesions are found to be interfering with fertility, a myomectomy, or surgical removal of the tumor and adhesions, can be scheduled. If the subfertility problem is identified as tubal insufficiency from inflammation, the prescription of diathermy or steroid administration may be helpful to reduce adhesions.

A postpartum woman is developing a thrombophlebitis in her right leg. Which assessments would the nurse make to detect this? 1. Bend her knee, and palpate her calf for pain 2. Assess for pedal edema. 3. Blanch a toe, and count the seconds it takes to color again. 4. Ask her to raise her foot and draw a circle

2. Assess for pedal edema. Calf swelling, erythema, warmth, tenderness, and pedal edema may be noted and are caused by an inflammatory process and obstruction of venous return.

The LPN has reported that uterine massage is ineffective on a client. The nurse anticipates the health care provider will prescribe which medication to address this issue? 1. Digoxin 2. Oxytocin 3. Ibuprofen 4. Penicillin

2. Oxytocin

When assessing the uterus of a 2-day postpartum client, which finding would the nurse evaluate as normal? 1. a scant amount of lochia serosa 2. a moderate amount of lochia rubra 3. a scant amount of lochia alba 4. a moderate amount of lochia alba

2. a moderate amount of lochia rubra The client should have lochia rubra for 3 to 4 days postpartum. The client would then progress to lochia serosa being expelled from day 3 to 10. Lastly, the client would have lochia alba from day 10 to 14 until 3 to 6 weeks.

A woman comes to the clinic because she has been unable to conceive. When reviewing the woman's history, the nurse would least likely identify which factor as a possible risk? 1. weight below standard for height and age 2. age of 25 years 3. history of smoking 4. diabetes since age 15 years

2. age of 25 years Female risk factors for infertility include increased age older, smoking and alcohol consumption, history of chronic illness such as diabetes, and overweight or underweight, which can disrupt hormonal function.

The nurse is providing teaching to a new mother who is breastfeeding. The mother demonstrates understanding of teaching when she identifies which characteristics as being true of the stool of breastfed newborns? Select all that apply. 1. firm in shape 2. yellowish gold color 3. stringy to pasty consistency 4. completely odorless 5. formed in consistency

2. yellowish gold color 3. stringy to pasty consistency

A school health nurse is providing education to a group of adolescents regarding the proper procedure for male condom use. The nurse knows the teaching has been effective when which statement is made by a student? 1. "It is important to put the condom on just before the penis is erect." 2. "Ensure your condom is always available, so store condoms in your wallet." 3. "Withdraw the penis erect, holding the condom firmly against the penis." 4. "Use only petroleum-based lubricants, such as body lotion or massage oil."

3. "Withdraw the penis erect, holding the condom firmly against the penis."

A client prescribed oral contraceptive pills (OCPs) has presented for a routine visit. Which finding, if reported by the client upon assessment, should the nurse prioritize? 1. light menstrual flow 2. cramping during menses 3. abdominal pain 4. small amount of breakthrough bleeding

3. abdominal pain The warning signs to report for a client on OCPs are severe abdominal or chest pain, dyspnea, headache, weakness, numbness, blurred or double vision, speech disturbances, or severe leg pain and edema. Light bleeding, light flow, and cramping are all normal.

The parent of a newborn notices swelling across the top of the newborn's head and is concerned. The parent states, "This looks painful. What can be done?" The nurse is correct to suggest which action? 1. A cool compress for the client to place on the site for 15 minutes at a time 2. Positioning the newborn in an upright position, which will eliminate the swelling 3. Measurement of the edematous area with health care provider notification 4. Continued assessment but no treatment needed as it will resolve on its own

4. Continued assessment but no treatment needed as it will resolve on its own Caput succedaneum is swelling that occurs typically across the presenting part of the head and often crosses a suture line. This occurs in cephalic births and is gradually absorbed and disappears without treatment. There is no need to place a cool compress, place the newborn in an upright position, or notify the health care provider of this condition.

The nurse is administering methylergonovine 0.2 mg to a postpartum client with uterine subinvolution. Which assessment will the nurse need to make prior to administering the medication? 1. if blood pressure is lower than 140/90 mm Hg 2. if the client can walk without experiencing dizziness 3. if hematocrit level is higher than 45% 4. if urine output is higher than 50 ml/h

if blood pressure is lower than 140/90 mm Hg

A mother just gave birth 3 hours ago. The nurse enters the room to continue hourly assessments and finds the client on the phone telling the listener about her fear while driving to the hospital and not making it in time. The mother finishes the call, and the nurse begins her assessment with which phrase? 1. "It sounded like you had quite a time getting here. Would you like to continue your story?" 2. "If you plan to breastfeed, you need to calm down." 3. "You have a beautiful baby, why worry about that now?" 4. "I need to assess your fundus now."

1. "It sounded like you had quite a time getting here. Would you like to continue your story?" The mother is going through the taking-in phase of relating events during her pregnancy and birth. The nurse can facilitate this phase by allowing the mother to express herself. Diverting the conversation, admonishing the mother, or warning of potential problems does not accomplish this facilitation.

The nurse has admitted a small-for-gestational-age infant (SGA) to the observation nursery from the birth room. Which action would the nurse prioritize in the newborn's care plan? 1. Closely monitor temperature. 2. Assess for hyperglycemia. 3. Monitor intake and output. 4. Observe feeding tolerance

1. Closely monitor temperature

A pregnant client has been diagnosed with gonorrhea. Which nursing interventions should be performed to prevent gonococcal ophthalmia neonatorum in the baby? 1. Instill a prophylactic agent in the eyes of the newborn. 2. Administer an antiretroviral syrup to the newborn. 3. Administer cephalosporins to mother during pregnancy. 4. Perform cesarean birth to prevent infection.

1. Instill a prophylactic agent in the eyes of the newborn.

The nurse is giving an educational presentation to the local Le Leche league chapter. One woman asks about risk factors for mastitis. Which condition would the nurse most likely include in the response? 1. Pierced nipple 2. Frequent feeding 3. Use of breast pumps 4. Complete emptying of the breast

1. Pierced nipples

The nurse notes a newborn has a temperature of 97.0oF (36.1oC) on assessment. The nurse acts to prevent which complication first? 1. Respiratory distress 2. Hypoglycemia 3. Cardiovascular distress 4. Seizure

1. Respiratory distress It takes oxygen to produce heat and an infant who has an episode of cold stress is at risk for respiratory distress. The infant needs to be warmed. The temperature should be in the range of 97.7°F to 98.6°F (36.5°C to 37°C). After respiratory distress sets in, it can be followed by seizures, cardiovascular distress, or hypoglycemia.

A pregnant woman in her 39th week of pregnancy presents to the clinic with a vaginal infection. She tests positive for chlamydia. What would this disease make her infant at risk for? 1. blindness 2. chickenpox 3. neonatal laryngeal papillomas 4. deafness

1. blindness

A woman in the 15th week of gestation is planning on terminating the pregnancy. The procedure done for second-trimester terminations is which of the following? 1. dilatation and extraction 2. dilatation and curettage 3. menstrual extraction 4. hysterotomy

1. dilatation and extraction Most second-trimester terminations (12 to 16 weeks' gestation) are done by dilatation and vacuum extraction. A dilatation and curettage are done when a pregnancy is less than 13 weeks. If the gestational age for a pregnancy is more than 16 weeks, a hysterotomy is done. A menstrual extraction is the simplest of all and is performed on the basis of 5 to 7 weeks after the last menstrual period.

A newborn is admitted to the nursery. Maternal history reveals the use of opioids. When assessing this newborn for symptoms of opioid withdrawal, which of the following does the nurse expect the newborn to exhibit first? 1. tremors 2. poor feeding 3. weight loss 4. diarrhea

1. tremors With opioid withdrawal, CNS symptoms appear first, followed by gastrointestinal disturbances. The most common symptom of neonatal abstinence syndrome is tremors.

The nurse is performing an assessment on a 2-day postpartum client and discovers a boggy fundus at the umbilicus and slightly to the right. The nurse determines that this is most likely related to which situation? 1. Full bowel 2. Bladder distention 3. Uterine atony 4. Poor bladder tone

2. Bladder distention Most often the cause of a displaced uterus is a distended bladder. Ask the client to void and then reassess the uterus. According to the scenario described, the most likely cause of the uterine findings would not be uterine atony. A full bowel or poor bladder tone would not cause a boggy and displaced fundus.

During a physical assessment of a newborn, the nurse observes bluish markings across the newborn's lower back. The nurse interprets this finding as: 1. stork bites. 2. congenital dermal melanocytosis (slate gray nevi). 3. birth trauma. 4. milia.

2. congenital dermal melanocytosis (slate gray nevi).

A postpartum woman who developed deep vein thrombosis is being discharged on anticoagulant therapy. After teaching the woman about this treatment, the nurse determines that additional teaching is needed when the woman makes which statement? 1. "I need to avoid drinking any alcohol." 2. "I will use a soft toothbrush to brush my teeth." 3. "I can take ibuprofen if I have any pain." 4. "I will call my health care provider if my stools are black and tarry."

3. "I can take ibuprofen if I have any pain."

A newborn is diagnosed with hemolytic disease of the newborn. When developing the plan of care for this child, the nurse would expect which of the following to be included as part of the treatment plan? 1. surfactant administration 2. radiant warming 3. exchange transfusion 4. mechanical ventilation

3. exchange transfusion Treatment for hemolytic disease of the newborn includes phototherapy and exchange transfusions. Surfactant is administered if the newborn has a surfactant deficiency. A radiant warmer is used to assist with thermoregulation. Mechanical ventilation is used for severe respiratory distress and for newborns with intraventricular hemorrhage.

A nurse is assessing a postpartum client who is at home. Which statement by the client would lead the nurse to suspect that the client may be developing postpartum depression? 1. "I keep hearing voices telling me to take my baby to the river." 2. "I just feel so overwhelmed and tired." 3. "It's strange, one minute I'm happy, the next I'm sad." 4. "I'm feeling so guilty and worthless lately."

4. "I'm feeling so guilty and worthless lately."

A nurse is preparing a presentation for a health fair on the topic of vasectomy. Which information should the nurse point out in the presentation? 1. Procedure is effective immediately 2. Birth control measures are not required 3. Regular sperm counts are not essential 4. Relatively easy procedure with few complications

4. Relatively easy procedure with few complications The nurse should inform the individuals that vasectomy is a relatively easy procedure with few complications. Regular sperm counts following a vasectomy are important, as it will take approximately a month for the remaining sperm to exit the body. The client should use birth control measures until his sperm count remains at zero for 6 weeks.

A woman visits the family planning clinic to request a prescription for birth control pills. Which factor would indicate that an ovulation suppressant would not be the best contraceptive method for her? 1. She has irregular menstrual cycles. 2. She has a history of allergy to foreign protein. 3. She is 30 years old. 4. She has a family history of thromboembolism.

4. She has a family history of thromboembolism. The estrogen content of birth control pills may lead to increased blood clotting, leading to an increased incidence of thromboembolism. Women who already are prone to this should not increase their risk further.

A premature infant develops respiratory distress syndrome. With this condition, circulatory impairment is likely to occur because, with increased lung tension: 1. there are aortic valve strictures. 2. the pulmonary artery closes. 3. the foramen ovale closes prematurely. 4. the ductus arteriosus remains open.

4. the ductus arteriosus remains open. Excess pressure in the alveoli stimulates the ductus arteriosus to remain open, compromising efficient cardiovascular function.

The nurse instructs a client on cervical mucus changes that occur during ovulation. Which statement indicates that teaching has been effective? 1. "When the mucus is thin and watery, then ovulation is occurring." 2. "Ovulation makes the mucus more acidic." 3. "The mucus is white because of more white blood cells." 4. "During ovulation, the mucus is thick."

1. "When the mucus is thin and watery, then ovulation is occurring." On the day of ovulation, the cervical mucus becomes copious, thin, watery, and transparent. During ovulation, the mucus is not thick, not acidic, and not white.

A nurse is caring for an infant with an elevated bilirubin level who is under phototherapy. What evaluation data would best indicate that the newborn's jaundice is improving? 1. Bilirubin level went from 15 to 11. 2. Reticulocyte count is 6%. 3. Skin looks less jaundiced. 4. Hematocrit is 38.

1. Bilirubin level went from 15 to 11. The newborn has physiologic jaundice, which is related to decreased bilirubin conjugation. Newborns have relatively immature livers and cannot conjugate (break down) bilirubin as fast as needed. Bilirubin overproduction is responsible for causing jaundice. A serum bilirubin is the best way to determine whether the jaundice is improving. The other listed methods will not address the needed information

An infant who is diagnosed with meconium aspiration displays which symptom? 1. intercostal and substernal retractions 2. no heart murmur 3. respirations of 45 4. pink skin

1. intercostal and substernal retractions Meconium aspiration is when the infant passes the first stool in utero and some of stool particles are ingested into the lungs at birth. This can cause the infant to be in distress displayed by mild cyanosis, tachypnea, retractions, hyperinflated chest, and hypercapnia.

In an effort to decrease complications for the infant right after birth, the nurse would expect to administer which medication for prophylaxis of potential eye conditions? 1. gentamicin ophthalmic ointment 2. erythromycin ophthalmic ointment 3. vitamin K 4. silver nitrate solution

2. erythromycin ophthalmic ointment

A nurse is asked to teach a woman to take her basal body temperature daily to assess the time of ovulation. She can detect her day of ovulation, following ovulation, because her temperature will: 1. decrease a degree. 2. increase a degree. 3. fluctuate a degree daily. 4. no longer reflect basal body temperature.

2. increase a degree. The effect of progesterone, released with ovulation, is to increase body temperature.

A couple comes to the clinic and states to the nurse, "I don't think we are ever going to be able to have children. We have been trying but have had no luck." What assessments does the nurse anticipate will be performed for this couple? Select all that apply. 1. in vitro fertilization counseling 2. ovulation monitoring 3. tubal patency 4. fertility drugs 5. semen analysis

2. ovulation monitoring 3. tubal patency 5. semen analysis Today, a subfertility investigation usually is limited to three assessments: semen analysis, ovulation monitoring, and tubal patency.

When planning care for a postpartum client, the nurse is aware that which site is the most common for postpartum infection? 1. within the blood stream 2. in the urinary bladder 3. in the reproductive tract 4. in the milk ducts

3. Reproductive tract The most common site for a postpartum infection is the reproductive tract. This is important for teaching and education of clients.

What important information should the nurse give a client about the use of a diaphragm during menstruation? 1. Use during menstruation can lead to abdominal pain. 2. It may cause genital burning. 3. Vaginal discharges may occur. 4. Toxic shock syndrome is possible.

4. Toxic shock syndrome is possible. The nurse should inform the client that using a diaphragm during menstruation can lead to toxic shock syndrome. Vaginal discharge is associated with the use of a vaginal contraceptive ring. Genital burning may occur when spermicidal products are used along with the barrier methods of contraception. Abdominal pain is a side effect reported during the use of oral contraceptive pills (OCPs) and intrauterine devices (IUDs).

A nurse is providing preoperative care to a female newborn client with the congenital abnormality myelomeningocele. Which intervention is the priority? 1. preserving newborn GI function 2. maximizing newborn motor function 3. promoting newborn nutrition 4. preventing infection

4. preventing infection A congenital condition of the newborn with a spinal deformity puts the newborn at risk for infection. A myelomeningocele is a fluid-filled sac on the spine that includes part of the spinal cord defect and the meninges. This cyst on the outside of the newborn requires surgical intervention. Although nutrition, GI function, and motor function are all important to the health of the newborn, the spinal and meninges defect puts the newborn at high risk for infection.

A nurse is to care for a client during the postpartum period. The client reports pain and discomfort in her breasts. What signs should a nurse look for to find out if the client has engorged breasts? Select all that apply. 1. Breasts are hard. 2. Breasts are tender. 3. Nipples are fissured. 4. Nipples are cracked. 5. Breasts are soft.

1. Breasts are hard. 2. Breasts are tender. Engorged breasts are hard and tender, and the nurse should assess for these signs. Improper positioning of the infant on the breast, not engorged breasts, results in cracked, blistered, fissured, bruised, or bleeding nipples in the breastfeeding woman.

A client needs additional information about the cervical mucus ovulation method after having read about it in a magazine. She asks the nurse about cervical changes during ovulation. What information should the nurse give the client? 1. Cervix is high or deep in the vagina. 2. Cervical mucus is dry and thick. 3. Cervical mucus breaks when stretched. 4. Cervical os is slightly closed.

1. Cervix is high or deep in the vagina. The nurse should explain that during ovulation, the cervix is high or deep in the vagina. The os is slightly open during ovulation. Under the influence of estrogen during ovulation, the cervical mucus is copious and slippery and can be stretched between two fingers without breaking. It becomes thick and dry after ovulation, under the influence of progesterone.

Which of the following would lead you to suspect that a newborn has developmental hip dysplasia? 1. continual drawing of his legs under him while prone 2. inability of the right hip to abduct 3. inward rotation of his right foot 4. crying on straightening of the right leg

2. inability of the right hip to abduct Newborns whose acetabulums are shallow cannot abduct their hip joint.

A woman calls the clinic to report that she has had some cramping and spotting since the insertion of her IUD three days ago. Which instruction would be most appropriate? 1. "Consider this normal, because your IUD is newly inserted." 2. "You'll have to change your method of birth control." 3. "Come to the clinic as soon as possible." 4. "Take your blood pressure daily for the rest of the month."

1. "Consider this normal, because your IUD is newly inserted." A woman may notice some spotting or uterine cramping the first 2 or 3 weeks after IUD insertion. Ibuprofen, a prostaglandin inhibitor, is helpful in relieving the pain.

A male client is being evaluated for subinfertility. What additional data should a nurse obtain? 1. testosterone, Rh factor, blood type, UA 2. testosterone, HIV status, CBC, sonogram 3. HIV status, testosterone, HCG, cholesterol 4. blood type, thyroid function, HCG, CBC

1. testosterone, Rh factor, blood type, UA The nurse should assess data related to testosterone levels, Rh factors for inflammation, blood type, and urinalysis.

A young woman says she needs a temporary contraceptive but has a latex allergy. She mentions that she has had a papillomavirus infection. Also, she says she is terrible about remembering to take pills. Which method should the nurse recommend? 1. transdermal contraception 2. sterilization 3. diaphragm 4. cervical cap

1. transdermal contraception The fact that this woman has a latex allergy rules out the cervical cap and diaphragm. Moreover, the diaphragm is contraindicated in her case due to her papillomavirus infection. The best choice for her is transdermal contraception, which involves wearing a patch for a week at a time and does not require taking pills daily.

A nurse is performing a newborn assessment and notes the blood pressures in the upper extremities are higher than the lower extremities. The nurse should suspect which congenital newborn abnormality? 1. ventricular septal defect 2. coarctation of the aorta 3. patent ductus arteriosus 4. truncus arteriosus

2. coarctation of the aorta In congenital heart defects, coarctation of the aorta occurs when there is a narrow or constricted area of the aorta. This causes blood pressures to be higher in the upper extremities and lower in the lower extremities. Patent ductus arteriosus refers to an open patent foramen ovale after birth, and a ventricular septal defect is an opening in the ventricle. Both of these latter disorders cause increased pulmonary flow in the heart. Truncus arteriosus means there is one main branch for all vessels coming off the top of the heart.

The nurse is caring for a 6-month-old infant who is the probable victim of abusive head trauma (shaken baby syndrome). The nurse is completing the baseline neurologic assessment. Which assessment finding requires health care provider notification because it is a sign of early increased intracranial pressure? 1. spitting up a mouthful of formula 2. irritability 3. pupils reactive to light and accommodation 4. positive Babinski sign

2. irritability Abusive head trauma (shaken baby syndrome) is the forceful shaking of an infant, which may cause damage to the brain or death. Infant irritability is an early sign of intracranial pressure. Spitting up a mouthful of milk or formula is normal in the infant. Reactive pupils and a positive Babinski sign are also normal findings.

A young woman comes to the free clinic asking for oral contraceptive pills. Which factor best indicates that another type of contraception would be better for this client? 1. frequency of sexual encounters 2. age of her partner 3. history of noncompliance with medications 4. cost of the pills

3. history of noncompliance with medications

The nurse instructs a client on actions to prevent postpartum depression. During a home visit, which observation indicates that instruction has been effective? 1. The client is cleaning the kitchen while the baby naps. 2. The client appears disheveled and listless. 3. The client complains of fatigue. 4. The client is chatting on the telephone with a friend.

4. The client is chatting on the telephone with a friend. Chatting on the phone with friends indicates that the client is not becoming isolated with baby care. This will help prevent the onset of postpartum depression. Fatigue, listlessness, and trying to be perfect with cleaning are observations that could indicate postpartum depression.

A nurse is counseling women on birth control choices. Which woman is the best candidate for an IUD? 1. a woman who has multiple sex partners 2. a woman who has a history of PID 3. a woman who is in her early twenties and wants children later in life 4. a woman who has one partner and three children

4. a woman who has one partner and three children A women who is in a stable monogamous relationship and has already had one successful pregnancy is the best candidate for an IUD. An IUD is not without risk to a woman. IUDs do not protect against STIs. PID may be caused by IUDs. A woman in her twenties has several better, lower risk options.

On a follow-up visit to the clinic, a nurse suspects that a postpartum client is experiencing postpartum psychosis. Which finding would most likely lead the nurse to suspect this condition? 1. sadness 2. feelings of anxiety 3. insomnia 4. delusional beliefs

4. delusional beliefs

A 12-hour-old infant is receiving IV fluids for polycythemia. For which complication should a nurse monitor this client? 1. hypotension 2. decreased level of consciousness 3. tachycardia 4. fluid overload

4. fluid overload The possibility of fluid overload is increased and must be considered by a nurse when administering IV therapy to a newborn. IV therapy does not significantly increase heart rate or change blood pressure, as well as the level of consciousness, unless fluid overload occurs.

A newborn is suspected to have fetal alcohol syndrome as a result of maternal use of alcohol during pregnancy. Which of the following would the nurse expect to assess? 1. thick upper lip 2. large bulging eyes 3. long nose 4. low nasal bridge

4. low nasal bridge A low nasal bridge is seen in fetal alcohol syndrome (FAS). The other features of FAS include thin, flat upper lip, small eyes with short palpebral fissure, flattened midface with a short nose. The facial features result from damage to the embryonic cells in early pregnancy.

A new mother of a newborn girl calls the clinic in a panic, concerned about the blood-tinged soiled diaper. What is the best response from the nurse? "The baby may have a problem; let's schedule an appointment." "This can be from the sudden withdrawal of your hormones. It is not a cause for alarm." "This can be related to cleaning her perineal area; be more careful." "If this continues, call us back; for now, just watch her."

"This can be from the sudden withdrawal of your hormones. It is not a cause for alarm." The mother is describing pseudomenstruation and is usually the result of the infant no longer having the mother's hormones in the body. This is not a cause for alarm. It is always appropriate to offer to schedule an appointment if the mother continues to be upset. The nurse should know that the infant's "bleeding" is not indicative of a pathologic process and should be careful to not upset the mother further. The statement of it being related to the way the mother is cleaning the perineum is incorrect for it places the blame on the mother for the infant's problem. The instruction to call back if it continues does not meet the mother's need to know why this is happening to her baby, and it negates her concern for her infant.

After teaching the parents of a newborn with retinopathy of prematurity (ROP) about the disorder and treatment, which statement by the parents indicates that the teaching was successful? 1. "Can we schedule follow-up vision screenings with the pediatric ophthalmologist now?" 2. "I'm sure the baby will grow out of it." 3. "We'll make sure to administer eye drops each day for the next few weeks." 4. "We can fix the problem with surgery."

1. "Can we schedule follow-up vision screenings with the pediatric ophthalmologist now?" Parents of a newborn with suspected retinopathy of prematurity (ROP) should schedule follow-up vision screenings with a pediatric ophthalmologist every 2 to 3 weeks, depending on the severity of the findings at the initial examination.

A nurse is providing care to a postpartum woman. The woman gave birth vaginally at 2 a.m. The nurse would anticipate the need to catheterize the client if she does not void by which time? 1. 9:00 a.m. 2. 3:30 a.m. 3. 5:15 a.m. 4. 7:45 a.m.

1. 9:00 am If a woman has not voided within 4 to 6 hours after giving birth, catheterization may be needed because a full bladder interferes with uterine contraction and may lead to hemorrhage. Not voiding by 9 a.m. exceeds the 4 to 6 hour time frame.

Which of the following would not be considered a disadvantage of the calendar/rhythm method of family planning? 1. cost 2. need for motivation 3. effectiveness 4. need for cooperation

1. cost The calendar/rhythm method requires no real cost to employ. The other options cited are all disadvantages.

Assessment of a pregnant client reveals that she has tested positive for a sexually transmitted infection (STI). The nurse understands that the client's newborn is at risk for which of the following? 1. neurologic damage 2. low birth weight 3. hypertension 4. diabetes

1. neurologic damage The nurse should inform the client that children born of mothers with STIs are often born with neurologic damage, congenital abnormalities, and meningitis. Children born of STI mothers are not known to be born with diabetes or hypertension. Low birth weight is associated with smoking during pregnancy.

The nurse completes instructing a female client on the process of in vitro fertilization. Which statement indicates that client teaching has been effective? 1. "Most procedures are effective the first time tried." 2. "It can be done with frozen donor sperm." 3. "I will need to select a surrogate mother." 4. "This is dangerous if there is ovarian cancer in my family."

2. "It can be done with frozen donor sperm." Fresh or frozen sperm can be used for in vitro fertilization. A surrogate mother is not needed for this procedure. Often, more than one attempt is needed before successful implantation occurs. The success of in vitro fertilization is not related to the incidence of ovarian cancer in the family.

The nurse in the NICU is caring for preterm newborns. Which guidelines are recommended for care of these newborns? Select all that apply. 1. Give the newborn a warm bath immediately. 2. Supply oxygen for the newborn, if necessary. 3. Discourage contact with parents to maintain asepsis. 4. Handle the newborn as much as possible. 5. Take the newborn's temperature often. 6. Dress the newborn in ways to preserve warmth.

2. Supply oxygen for the newborn, if necessary. 5. Take the newborn's temperature often. 6. Dress the newborn in ways to preserve warmth. Controlling the temperature of preterm newborns is often difficult; therefore, special care should be taken to keep these babies warm. Nurses should dress them in a stockinette cap, take their temperature often, and supply oxygen, if necessary. To conserve the energy of small newborns, the nurse should handle them as little as possible. Usually, they will not give them a bath immediately. Parents should be encouraged to bond with their infants.

The nursing instructor is conducting a class explaining the various causes of jaundice in a newborn infant. The instructor determines additional education is warranted after the class chooses which factor as being responsible for newborn jaundice? 1. impaired bilirubin excretion 2. bilirubin hyperexcretion 3. bilirubin overproduction 4. decreased bilirubin conversion

2. bilirubin hyperexcretion Overexcretion of bilirubin would not cause jaundice. Bilirubin overproduction, decreased bilirubin conjugation or conversion, and impaired bilirubin excretion would cause hyperbilirubinemia, which leads to jaundice.

All of the following are signs of infection in a preterm newborn except: 1. glucose instability. 2. hyperreflexia. 3. temperature instability. 4. diarrhea.

2. hyperreflexia. Temperature and glucose instability are signs of infection in a preterm newborn. Diarrhea is also a sign. Preterm newborns with infection usually present with hypotonia, not with hyperreflexia.

The nurse is concerned that a new parent is developing a postpartum complication. What did the nurse most likely assess in this client? 1. red-colored lochia for the first 24 hours 2. lochia that has an offensive odor 3. lochia appearing pinkish-brown on the fourth day 4. lochia that is the color of menstrual blood

2. lochia that has an offensive odor Lochia should not have an offensive odor, because this suggests the uterus has become infected. Immediate intervention is needed to halt postpartum infection. Red-colored lochia for the first 24 hours is normal. Lochia that is the color of menstrual blood is normal. Lochia appearing pinkish-brown on the fourth postpartum day is normal.

A nursing instructor is teaching about subfertility to nursing students and perceives their understanding when they define subfertility existing when a pregnancy has not occurred after how much time of engaging in unprotected sex? 1. 18 months 2. 6 months 3. 12 months 4. 9 months

3. 12 months Subfertility is said to exist when a pregnancy has not occurred after at least 1 year of engaging in unprotected sex.

Assessment of a 26-week-old premature newborn reveals that the newborn is having problems with thermoregulation. The nurse would be alert for the development of which of the following? 1. tachycardia 2. sleepiness 3. apnea 4. crying

3. Apnea A premature infant has thin skin, an immature central nervous system (CNS), lack of brown fat stores, and an increased weight-to-surface area ratio. These are predisposing thermoregulation problems that can lead to hypothermia. As a result, the infant may become apneic, have respiratory distress, increase his or her oxygen need, or be cyanotic. Tachycardia, sleepiness, and crying are unrelated to thermoregulatory problems.

A clinic nurse explains to a client who is undergoing an infertility workup that the patency of her fallopian tubes will be checked. Which test is currently used to do this? 1. uterine biopsy 2. magnetic resonance imaging 3. hysterosalpingography 4. computed tomography scan

3. hysterosalpingography Hysterosalpingography is radiologic examination with radiopaque contrast of the fallopian tubes. It is widely used to assess the patency of the fallopian tubes in women who are subfertile.

Which description best explains the hysterosalpingogram procedure? 1. insertion of an endoscope through the posterior fornix to visualize the reproductive organs 2. passage of an endoscope through a small abdominal incision to inspect the reproductive organs 3. radiograph of the uterus and fallopian tubes following introduction of a radiopaque medium through the cervix 4. instillation of carbon dioxide through the cervix into the uterus and fallopian tubes

3. radiograph of the uterus and fallopian tubes following introduction of a radiopaque medium through the cervix Hysterosalpingogram is a procedure to document the patency of the fallopian tubes through the use of a radiopaque medium.

A nurse is conducting a review class for a group of perinatal nurses working at the local clinic. The clinic sees a high population of women who are HIV positive. After discussing the recommendations for antiretroviral therapy with the group, the nurse determines that the teaching was successful when the group identifies which rationale as the underlying principle for the therapy? 1. treatment of opportunistic infections 2. can cure acute HIV/AIDS infections 3. adjunct therapy to radiation and chemotherapy 4. reduction in viral loads in the blood

4. reduction in viral loads in the blood Drug therapy is the mainstay of treatment and is important in reducing the viral load as much as possible. Antiretroviral agents do not treat opportunistic infections and are not adjunctive therapy. There is no cure for HIV/AIDS.

The mother of a preterm infant tells her nurse that she would like to visit her newborn in the neonatal intensive care unit (NICU). Which response by the nurse would be most appropriate? 1. "Certainly. You will need to wash your hands and gown before you can hold him, however." 2. "Certainly. You may only observe the child from a distance, however, as his immune system is still not developed adequately." 3. "I'm sorry. You may not visit your son until he has been released from the NICU." 4. "I'm sorry. You may not visit the NICU, but we can arrange to have your son brought to your room so that you can hold him."

1. "Certainly. You will need to wash your hands and gown before you can hold him, however." The nurse should be certain the parents of a high-risk newborn are kept informed of what is happening with their child. They should be able to visit the special nursing unit to which the newborn is admitted as soon and as often as they choose, and, after washing and gowning, hold and touch their newborn. Both actions will help make the child's birth more real to them.

Which question would be most appropriate for a nurse to ask a client to assist in establishing a nursing diagnosis of Deficient knowledge related to measure to promote fertility? 1. "How often do you and your partner engage in intercourse?" 2. "How long have you not been able to get pregnant?" 3. "Have you ever been diagnosed with an STI?" 4. "How many sexual partners have you had in your life?"

1. "How often do you and your partner engage in intercourse?" An initial question that will gauge the knowledge level of the client related to reproduction is how frequently she engages in intercourse.

The pregnant woman with diabetes asks the nurse why her last baby weighed 11 pounds. What is the best response by the nurse? 1. "The fetus increases insulin production in response to elevated glucose levels of the mother, which acts as a fetal growth hormone." 2. "There is no way to control the amount of glucose the mother is producing, because she can't take insulin while she is pregnant and the baby gains too much weight." 3. "Your baby weighed so much because of how you were eating. You must eat less with this child." 4. "The fetus maintains elevated levels of glucose in response to the mother's eating patterns and gains too much weight."

1. "The fetus increases insulin production in response to elevated glucose levels of the mother, which acts as a fetal growth hormone." Consistently elevated fetal insulin levels cause the distinctive growth pattern. Because maternal glucose levels are elevated and glucose readily crosses the placenta, the fetus responds by increasing insulin production. Because insulin acts as a fetal growth hormone, consistently high levels cause fetal macrosomia, birth weight of greater than 4,500 g. Insulin also causes disproportionate fat buildup to the shoulders and upper body, increasing the risk for shoulder dystocia and birth trauma.

In pulse oximetry for a newborn, what is the percentage of oxygen that is considered abnormal? 1. 75% 2. 95% 3. 87% 4. 85%

1. 75% Oxygen saturations below 80% are considered abnormal in a newborn. This indicates that the blood has poor oxygen content.

A nurse is providing care to a large-for-gestational-age newborn. The newborn's blood glucose level was 32 mg/dl one hour ago. Breastfeeding was initiated. The nurse checks the newborn's blood glucose level and finds it to be 23 mg/dl. Which action would the nurse do next? 1. Administer intravenous glucose. 2. Place the newborn under a radiant warmer. 3. Feed the newborn 2 ounces of formula. 4. Initiate blow-by oxygen therapy.

1. Administer intravenous glucose. Supervised breastfeeding or formula feeding may be the initial treatment options in asymptomatic hypoglycemia. However, symptomatic hypoglycemia should always be treated with frequent breast or formula feedings or dextrose gel massaged into the buccal mucosa. If hypoglycemia persists, then intravenous dextrose may be needed. Oral feedings would be used to maintain the newborn's glucose level above 40 mg/dl. Blow-by oxygen would have no effect on glucose levels; it may be helpful in promoting oxygenation. Placing the newborn under a radiant warmer would be a more appropriate measure for cold stress.

The nurse is providing discharge instructions to a client who has had a vasectomy. Which instruction is essential? 1. Continue to use a method of birth control. 2. Use a mild, over-the-counter analgesic for pain. 3. Return for a postoperative visit in 2 weeks. 4. Apply heat to the scrotum to promote healing.

1. Continue to use a method of birth control. Although the vasectomy is 99.5% effective in preventing pregnancy, spermatozoa can remain viable in the vas deferens for up to 6 months. Therefore, a client can resume sexual intercourse within 1 week but will need to continue to use an additional birth control method. Returning for the postoperative visit and taking a mild analgesic are important but not as important as preventing pregnancy. The client should use ice on the operation site, not heat.

The nurse is caring for a client at the ambulatory care clinic who questions the nurse for information about contraception. The client reports that she is not comfortable about using any barrier methods and would like the option of regaining fertility after a couple of years. Which method should the nurse suggest to this client? 1. CycleBeads or medroxyprogesterone injection 2. lactation amenorrhea method 3. coitus interruptus 4. basal body temperature (BBT)

1. CycleBeads or medroxyprogesterone injection The best option for a client who is not well educated would be the Standard Days Method with CycleBeads, as the 32 color-coded CycleBeads are easy to use and understand. An injection of medroxyprogesterone would also suit this client, as it works by suppressing ovulation and the production of follicle stimulating hormone and luteinizing hormone by the pituitary gland and prevents pregnancy for 3 months at a time. BBT requires the client to take and chart her body temperature; this may be difficult for the client to follow. Coitus interruptus is a method in which the man controls his ejaculation and ejaculates outside the vagina; this suggests that the client rely solely on the cooperation and judgment of her spouse. The lactation amenorrhea method works as a temporary method of contraception only for breastfeeding mothers.

A nurse is caring for an infant born with polycythemia. Which intervention is most appropriate when caring for this infant? 1. Focus on decreasing blood viscosity by increasing fluid volume. 2. Focus on monitoring and maintaining blood glucose levels. 3. Repeat screening every 2 to 3 hours or before feeds. 4. Check blood glucose within 2 hours of birth by reagent test strip.

1. Focus on decreasing blood viscosity by increasing fluid volume. The nurse should focus on decreasing blood viscosity by increasing fluid volume in the newborn with polycythemia. Checking blood glucose within 2 hours of birth by a reagent test strip and screening every 2 to 3 hours or before feeds are not interventions that will alleviate the condition of an infant with polycythemia. The nurse should monitor and maintain blood glucose levels when caring for a newborn with hypoglycemia, not polycythemia.

A client at 34 weeks' gestation has recently been diagnosed with human immunodeficiency virus (HIV). The client asks how HIV would be transmitted to the newborn. Which statement would be the nurse's best response? 1. It is recommended to formula-feed your newborn as it is transmitted through your breast milk." 2. "It is only transmitted through the birth canal so a cesarean birth will be scheduled." 3. "The risk of your newborn being infected with HIV infection is about 1%." 4. "It is not transmitted to your newborn as it is protected in the uterus."

1. It is recommended to formula-feed your newborn as it is transmitted through your breast milk." An infected mother can transmit HIV infection to her newborn before or during birth and through breastfeeding. The risk of perinatal transmission of HIV from an infected mother to her newborn is about 25%. This risk falls to less than 1% if the mother receives antiretroviral therapy during pregnancy. HIV can be spread to the infant through breastfeeding. HIV-infected mothers should be counseled to avoid breastfeeding and use formula instead.

A nurse is providing care to a 34-week newborn diagnosed with neonatal abstinence syndrome (NAS). What is most important for the nurse to consider for successful management of the newborn with NAS? 1. Nursing care should be supportive and client specific. 2. Collaboration with social services is important 3. Pharmacologic therapy is preferred for newborns with NAS 4. Parents should be encouraged to participate in care

1. Nursing care should be supportive and client specific. The most important thing for the nurse to consider is that it is essential for care to be supportive and specific to the newborn. Parents should be encouraged to participate in care when possible and the nurse should collaborate with social services but it is most important that the nurse provide care that is specific to the newborn. Most newborns diagnosed with NAS do not require pharmacologic therapy and are treated with supportive measures until they finish withdrawal.

The parents of a 2-day-old newborn are preparing for discharge from the hospital. Which teaching is most important for the nurse to include regarding sleep? 1. Place the infant on the back when sleeping. 2. Caregivers need to sleep while the baby is sleeping. 3. Newborns usually sleep for 16 or more hours each day. 4. The infant may sleep through the night around 2 months of age.

1. Place the infant on the back when sleeping It is most important to educate caregivers on how to place the newborn while sleeping to ensure safety and reduce the risk of SIDS. The other information is good to include, but not priority.

A nurse supervisor observes a nurse massage a client's injection site after giving a dose of depot medroxyprogesterone acetate (Depo-Provera). What is the priority response by the nurse supervisor? 1. Remind the nurse that this injection should absorb slowly. 2. Call the health care provider immediately. 3. Thank the nurse for excellent client care. 4. Write up an incident report for the medication error.

1. Remind the nurse that this injection should absorb slowly. The medication should absorb slowly from the muscle, so the nurse supervisor should remind the nurse not to rub the injection site following administration.

A newborn with esophageal atresia has just returned from surgery to place a gastrostomy tube. Which client outcome will the nurse use when planning postoperative nursing care? 1. The client will remain free from infection at the gastrostomy tube site. 2. The client will cough and deep breathe following surgical procedure. 3. The client will have balanced nutrition within 4 weeks. 4. The client will reverse fluid volume excess within 2 days.

1. The client will remain free from infection at the gastrostomy tube site. An outcome of nursing care is important to identify early. For this newborn, the priority is skin integrity and the outcome is that the client will remain free from infection. Acidic gastric secretions can leak onto the skin from the gastrostomy site, leading to skin irritation. The nurse should plan interventions to protect the skin by using a cream or commercial skin protection system or consult with a wound, ostomy, and continence therapy nurse to reduce the possibility of skin irritation and infection. With the placement of the gastrostomy tube, the newborn is at less risk for imbalanced nutrition and excess fluid volume. The gastrostomy tube will not affect the newborn's gas exchange; however, a newborn is unable to complete coughing and deep breathing exercises.

A nurse is assigned to care for a newborn with hyperbilirubinemia. The newborn is relatively large in size and shows signs of listlessness. What most likely occurred? 1. The infant's mother probably had diabetes. 2. The infant may have been exposed to alcohol during pregnancy. 3. The infant may have experienced birth trauma. 4. The infant's mother must have had a long labor.

1. The infant's mother probably had diabetes. The nurse should know that the infant's mother more than likely had/has diabetes. The large size of the infant born to a mother with diabetes is secondary to exposure to high levels of maternal glucose crossing the placenta into the fetal circulation. Common problems among infants of mothers with diabetes include macrosomia, respiratory distress syndrome, birth trauma, hypoglycemia, hypocalcemia and hypomagnesemia, polycythemia, hyperbilirubinemia, and congenital anomalies. Listlessness is also a common symptom noted in these infants. Infants born to clients who use alcohol during pregnancy, infants who have experienced birth traumas, or infants whose mothers have had long labors are not known to exhibit these particular characteristics, although these conditions do not produce very positive pregnancy outcomes. Infants with fetal alcohol spectrum disorder or alcohol exposure during pregnancy do not usually have hypoglycemia problems.

Which safety precautions should a nurse take to prevent infection in a newborn? Select all that apply. 1. Use sterile gloves for an invasive procedure. 2. Avoid using disposable equipment. 3. Cover jewelry while washing hands. 4. Initiate universal precautions when caring for the infant. 5. Avoid coming to work when ill.

1. Use sterile gloves for an invasive procedure. 4. Initiate universal precautions when caring for the infant. 5. Avoid coming to work when ill. To minimize the risk of infections, the nurse should avoid coming to work when ill, use sterile gloves for an invasive procedure, and initiate universal precautions. The nurse should remove all jewelry before washing hands, not cover the jewelry. The nurse should use disposable equipment rather than avoid it.

The newborn nursery nurse suspects a newborn of having neonatal abstinence syndrome. What assessment findings would most correlate with the diagnosis? 1. frequent yawning and sneezing 2. positive Babinski and Moro reflexes 3. vigorous rooting and feeding 4. cyanotic discoloration of the hands and feet

1. frequent yawning and sneezing Manifestations of neonatal abstinence syndrome include: CNS dysfunction such as hyperactive reflexes resulting in exaggerated Babinski and Moro reflexes; hypertonic muscle tone and constant movement; metabolic, vasomotor, and respiratory disturbances with frequent yawning and sneezing; gastrointestinal dysfunction, including poor feeding; and frantic sucking or rooting. Acrocyanosis is a normal newborn finding which is cyanotic discoloration of the extremities.

A client is beginning to take oral contraceptive pills (OCPs). Which side effects will the nurse caution might be expected? Select all that apply. 1. headache 2. weight loss 3. breast tenderness 4. nausea 5. frequent urinary tract infections

1. headache 3. breast tenderness 4. nausea The main side effects that women may experience are nausea, weight gain, headache, breast tenderness, breakthrough bleeding, monilial vaginal infections, mild hypertension, and depression.

A nurse is caring for a 25-year-old G1P0 at 37 weeks' gestation. The client's history indicates that the client has had alcohol abuse disorder throughout their pregnancy. What signs and symptoms does the nurse expect the newborn to exhibit? 1. high pitched, shrill cry 2. jitterness 3. large narrow spaced eyes 4. increased appetite 5. thin upper lip

1. high pitched, shrill cry 2. jitterness 5. thin upper lip Fetal alcohol syndrome (FAS) is caused by intake of alcohol by the pregnant parent during pregnancy; alcohol consumption may be periodic or chronic. Newborns born with FAS have characteristic facial features, are more susceptible to congenital defects, and often have developmental delays. Newborns with FAS have a high-pitched and shrill cry and are generally jittery. Newborns with FAS have a flat midface and a thin upper lip. Newborns with FAS have small and wide-spaced eyes not large narrow-spaced eyes. Newborns with FAS are not easily consoled and have a poor, not an increased appetite

The nurse is providing contraception counseling to a perimenopausal woman who has had negative reactions to oral contraceptives (OCs) in the past and would like a long-term, nonhormone-based method that has a high rate of success. Neither she nor her husband wants to undergo surgery, however. Which method should the nurse recommend? 1. intrauterine device 2. subdermal progestin implant 3. tubal ligation 4. transdermal patch

1. intrauterine device Women who are perimenopausal are, overall, good candidates for intrauterine devices (IUDs). In this case, the IUD is the best choice because this method is almost 100% effective, is long-term, and does not involve any hormones. Both the transdermal patch and the subdermal progestin implant involve hormones, which rules each out in this case. Also, tubal ligation is a surgical procedure, which also rules it out for this client.

At 40 weeks' gestation and after 26 hours of labor a woman gives birth by cesarean section to a neonate weighing 4550 grams. The nurse implements which standard of care for this infant? 1. macrosomic infant 2. post-mature infant 3. NICU infant 4. hypoglycemic infant

1. macrosomic infant A birth weight of 4500 grams or greater classifies the infant as macrosomic. A macrosomic infant is at risk for hypothermia, hypoglycemia, and other complications. Therefore, close monitoring is needed. A post-mature infant is one born after 41 weeks' completed gestation. There is no evidence that this infant is hypoglycemic (low blood glucose level). Based on the data provided, there is no evidence of the need for NICU care.

For which client assessment finding would an intrauterine device (IUD) be contraindicated? 1. misshapen uterus 2. multiple sexual partners 3. history of thromboembolic disease 4. diagnosis of hypertension

1. misshapen uterus Use of an IUD may be contraindicated for a woman whose uterus is distorted in shape because the device might perforate the uterine wall. The device is not contraindicated for women with multiple sexual partners, women who have hypertension, or those with a history of thromboembolic disease. Infection is no longer a concern because the vaginal string no longer conducts fluid. The device does not impact hormone levels and will not influence blood pressure or blood flow.

While providing care, the nurse suspects that a preterm infant is developing respiratory distress. What did the nurse most likely assess in this client? Select all that apply. 1. nasal flaring 2. oxygen saturation 96% 3. increasing respiratory rate 4. grunting 5. intercostal retractions

1. nasal flaring 3. increasing respiratory rate 4. grunting 5. intercostal retractions A steadily increasing respiratory rate, grunting, and nasal flaring are often the first signs of obstruction or respiratory compromise in newborns. If these are present, undress the baby's chest and look for intercostal retractions, which reflect the degree of difficulty the newborn is having in drawing in air. Oxygen saturation of 96% is within normal limits and does not indicate respiratory distress.

A newborn, born at 33 weeks' gestation, is on a ventilator in the neonatal intensive care unit (NICU). The newborn receives surfactant therapy. Which would the nurse expect to assess as a positive response to this therapy? 1. oxygen saturation 98% 2. partial pressure of carbon dioxide (PaCO2) 48 mm Hg 3. glucose 60 mg/dl (3.3 mmol/l) 4. heart rate 60 beats/min

1. oxygen saturation 98% Rescue treatment is indicated for newborns with established respiratory distress syndrome who require mechanical ventilation and supplemental oxygen. The earlier the surfactant is administered, the better the effect on gas exchange with an aim to have the oxygen saturation level of 98%. Glucose level assessment does not correlate with this therapy. The heart rate of 60 beats/min is an abnormal finding and not a positive result of the therapy. The PaCO2 greater than 45 mm Hg indicates respiratory acidosis. The normal value should be from 35 to 45 mm Hg.

The nurse is assessing a small-for-gestational age (SGA) newborn, 12 hours of age, and notes the newborn is lethargic with cyanosis of the extremities, jittery with handling, and a jaundiced, ruddy skin color. The nurse expects which diagnosis as a result of the findings? 1. polycythemia 2. hypercalcemia 3. hyperglycemia 4. hyponatremia

1. polycythemia Newborns born small for gestational age (SGA) are at risk for polycythemia. They should therefore undergo screening at 2, 12, and 24 hours of age. Observe for clinical signs of polycythemia (respiratory distress, cyanosis, jitteriness, jaundice, ruddy skin color, and lethargy).

A preterm infant is placed on ventilatory assistance for respiratory distress syndrome. In light of her lung pathology, which additional ventilatory measure would you anticipate planning? 1. positive end-expiratory pressure to increase oxygenation 2. administration of chilled oxygen to reduce lung spasm 3. increased inspiratory pressure; decreased expiratory pressure 4. administration of dry oxygen to avoid over-humidification

1. positive end-expiratory pressure to increase oxygenation Positive end-expiratory pressure, like expiratory grunting, prevents alveoli from fully closing on expiration and reduces the respiratory effort needed for inspiration.

A 26-year old mother of three children tells the nurse at her annual check-up that she does not want to have any more children; however, her religion forbids the use of contraceptive devices. Which of the following methods would be the best option for this client? 1. rhythm method 2. diaphragm 3. hormonal method 4. intrauterine device

1. rhythm method Fertility awareness methods (FAMs) include the rhythm method, which involves limiting sexual intercourse to the time during the woman's menstrual cycle when she is most likely to be infertile. This method does not involve contraceptive devices. Barrier methods such as diaphragms and IUDs interfere with conception by physically preventing sperm from fertilizing ova. Barriers work through mechanical and chemical means. Hormonal methods of birth control alter a woman's normal hormone level to prevent ovulation and thus the chances for conception.

Which sign appears early in a neonate with respiratory distress syndrome? 1. tachypnea more than 60 breaths/minute 2. bilateral crackles 3. poor capillary filling time (3 to 4 seconds) 4. pale gray skin color

1. tachypnea more than 60 breaths/minute Tachypnea and expiratory grunting occur early in respiratory distress syndrome to help improve oxygenation. Poor capillary filling time, a later manifestation, occurs if signs and symptoms aren't treated. Crackles occur as the respiratory distress progressively worsens. A pale gray skin color obscures earlier cyanosis as respiratory distress symptoms persist and worsen.

The nurse is caring for a large-for-gestational-age infant born to a client with diabetes mellitus. Why should the nurse schedule routine blood glucose measurements for the infant? 1. to detect rebound hypoglycemia 2. to determine insulin dosage to administer 3. to explain the effects of maternal hyperglycemia on the baby 4. to estimate the amount of calories to provide the infant through formula

1. to detect rebound hypoglycemia Large-for-gestational age infants need to be carefully assessed for hypoglycemia in the early hours of life because large infants require large amounts of nutritional stores to sustain their weight. If the mother had diabetes that was poorly controlled, the infant would have had an increased blood glucose level in utero to match the mother's glucose level; this caused the infant to produce elevated levels of insulin. After birth, these increased insulin levels will continue for up to 24 hours of life, possibly causing rebound hypoglycemia. Frequent blood glucose monitoring in large-for-gestational-age infants is not done to determine insulin dosage, to explain the effects of maternal hyperglycemia on the baby, or to estimate the amount of calories to provide the infant through formula.

A postpartum client who had a cesarean birth reports right calf pain to the nurse. The nurse observes that the client has nonpitting edema from her right knee to her foot. The nurse knows to prepare the client for which test first? 1. venous duplex ultrasound of the right leg 2. noninvasive arterial studies of the right leg 3. venogram of the right leg 4. transthoracic echocardiogram

1. venous duplex ultrasound of the right leg

A pregnant client has tested positive for hepatitis B virus. When discussing the situation with the client, the nurse explains that her newborn will be vaccinated with an initial HBV vaccine dose at which time? 1. within 12 hours of birth 2. within 24 hours of birth 3. within 36 hours of birth 4. within 48 hours of birth

1. within 12 hours of birth If a woman tests positive for HBV, the newborn will receive HBV vaccine within 12 hours of birth. The second dose will be given at 1 month and the third dose at 6 months.

An 18-year-old female client who is sexually active asks the nurse for information on barrier contraceptives. Which statement is appropriate for the nurse to include in the response? 1. "The use of a diaphragm is a good choice if you have frequent urinary tract infections." 2. "Cervical caps are most effective when used along with a spermicidal agent." 3. "A diaphragm must be inserted 1 hour prior to sexual intercourse." 4. "A cervical cap is a barrier contraceptive that you can purchase over the counter."

2. "Cervical caps are most effective when used along with a spermicidal agent." The nurse would include information on using spermicidal agents along with barrier contraceptives. A cervical cap is a mechanical barrier contraceptive that interferes with conception by physically preventing sperm from fertilizing ova. Cervical caps cannot be purchased over the counter, because the client must be properly fitted for the device to work effectively. Frequent urinary tract infections are a contraindication to the use of diaphragms because diaphragm use can place pressure placed on the urinary tract. Diaphragms can be placed up to 18 hours before intercourse; however, 6 hours is the most commonly recommended time. Spermicide should not be placed more than 2 hours before intercourse.

The nurse is conducting a health history on a couple planning to become pregnant over the next few months. Which statement by the male client most concerns the nurse? 1. "I have severe asthma and use my inhaler several times each month." 2. "I have a degree as a chemical engineer and work at the local plant." 3. "I consume fast food a few times each week and enjoy a few beers on the weekends." 4. "My uncle and his wife have not been able to conceive a child."

2. "I have a degree as a chemical engineer and work at the local plant." The client works around chemicals that could cause infertility issues. The nurse should further question the client as to what types of chemicals he is exposed to and what type of precautions are taken to protect him. Eating fast food and occasionally consuming alcohol are not risk factors for infertility. The client does not state a direct family member having issues with conception. An uncle is not concerning; however, a brother would be of concern. The client's asthma status should be monitored; however, this is not an issue regarding fertility.

A woman's baby is HIV positive at birth. She asks the nurse if this means the baby will develop AIDS. Which statement would be the nurse's best answer? 1. "HIV is transmitted at birth; having a cesarean birth prevented transmission." 2. "The antibodies may be those transferred across the placenta; the baby may not develop AIDS." 3. "HIV antibodies do not cross the placenta; this means the baby will develop AIDS." 4. "She already has AIDS. That's what being HIV positive means."

2. "The antibodies may be those transferred across the placenta; the baby may not develop AIDS." Infants born of HIV-positive women test positive for HIV antibodies at birth because these have crossed the placenta. An accurate disease status cannot be determined until the antibodies fade at about 18 months. Testing positive for HIV antibodies does not mean the infant has AIDS. Having a cesarean birth does decrease the risk of transmitting the virus to the infant at birth; it does not prevent the transmission of the disease. HIV antibodies do cross the placenta, which is why babies born of HIV positive mothers are HIV positive.

A client is seen in the clinic requesting an emergency postcoital contraception pill levonorgestrel (Plan B). The client states that she had unprotected sex 3 ½ days ago. What is the most appropriate response by the nurse? 1. "You should contact the local abortion clinic to set up an appointment." 2. "Unfortunately you are past the time frame to take that prescription." 3. "You will need to also be treated with antibiotics for an STI." 4. "The pill will cause you to begin menstruating within a day."

2. "Unfortunately you are past the time frame to take that prescription." Levonorgestrel can be taken anytime within 72 hours (3 days) of unprotected sexual activity. There is a newer form of the pill, ulipristal acetate, that can be taken as late as 120 hours after unprotected intercourse, but this medication does require a prescription.

A 38-year-old client wishes to discontinue use of oral contraceptive pills (OCPs) and attempt to conceive. The client says to the nurse, "I know a lot of women who have waited later to have children and became pregnant easily." How will the nurse respond? 1. "As soon as you stop taking the pill, you should be able to conceive. There are no age-related problems with you becoming pregnant." 2. "Women in their late 30s who use oral contraceptive pills may face a delay in conception, as the body takes a few months to be able to conceive." 3. "The age of a woman does not matter when trying to conceive, but you have been on birth control for a long time, thus it may take you longer." 4. "You should be off of oral contraceptive pills for at least 1 year before trying, and then you may or may not be able to become pregnant."

2. "Women in their late 30s who use oral contraceptive pills may face a delay in conception, as the body takes a few months to be able to conceive." Age impacts the ability to conceive. The chance of subfertility increases with age. Because of this gradual decline in fertility, women who defer pregnancy to their late 30s are apt to have more difficulty conceiving than their younger counterparts. Clients using oral, injectable, or implanted hormones for contraception may have difficulty becoming pregnant for several months after discontinuing these medications as the body needs time to restore normal functioning.

The nurse notes that a client's uterus, which was firm after the fundal massage, has become boggy again. Which intervention would the nurse do next? 1. Use semi-Fowler position to encourage uterine drainage. 2. Check for bladder distention, while encouraging the client to void. 3. Offer analgesics prescribed by health care provider. 4. Perform vigorous fundal massage for the client.

2. Check for bladder distention, while encouraging the client to void If the nurse finds a previously firm fundus to be relaxed, displaced, and boggy, the nurse should assess for bladder distention and encourage the woman to void or initiate catheterization as indicated. Emptying a full bladder facilitates uterine contraction and decreased bleeding. The nurse should not perform vigorous fundal massage. Excessive massage leads to overstimulation of uterine muscle, resulting in excessive bleeding. The nurse should place the client in a semi-Fowler position to encourage uterine drainage in the client with postpartum endometritis. The nurse should offer analgesics as prescribed by the health care provider to minimize perineal discomfort in clients experiencing postpartum lacerations.

A preterm infant has an umbilical vessel catheter inserted so that blood can be drawn readily. Which would be most important to implement during this procedure? 1. Prevent the infant from crying. 2. Ensure that the infant is kept warm. 3. Assess the infant's cranial vascular tension. 4. Evaluate the infant's urinary output.

2. Ensure that the infant is kept warm. Preterm infants must be protected from chilling during all procedures, because maintaining warmth is a major concern because of immaturity.

A nurse identifies the urethral opening of a male newborn to be on the dorsum of the shaft of the penis. How does the nurse document this finding? 1. Hydrocephalus 2. Epispadias 3. Talipes equinovarus 4. Omphalocele

2. Epispadias The newborn has epispadias. Hypospadias and epispadias are congenital anomalies in which the urethral opening is malpositioned. Urethral placement may be anywhere along the glands or shaft of the penis. In epispadias, the urethra opens on the dorsal aspect of the penis. In hypospadias, the urethra opens on the ventral aspect of the penis. Talipes equinovarus is a congenital skeletal deformity with the foot in a downward and inward flexed position. Hydrocephalus is an anomaly with infants having an abnormally large head. Omphalocele is a congenital anomaly of the abdominal wall, in which the abdominal contents contained within the peritoneal sac protrude through the external abdominal surface at the base of the umbilical cord.

What is the best rationale for trying to decrease the incidence of cold stress in the neonate? 1. The neonate will stabilize his or her temperature by 8 hours after birth if kept warm and dry. 2. If the neonate becomes cold stressed, he or she will eventually develop respiratory distress. 3. Evaporative heat loss happens when the neonate is not bundled and does not have a hat on. 4. It takes energy to keep warm, so the neonate has to remain in an extended position.

2. If the neonate becomes cold stressed, he or she will eventually develop respiratory distress. If cold stressed, the infant eventually will develop respiratory distress; oxygen requirements rise (even before noting a change in temperature), glucose use increases, acids are released into the bloodstream, and surfactant production decreases bringing on metabolic acidosis. A flexed position, not an extended position, keeps the neonate warm.

The nurse assesses the client who is 1 hour postpartum and discovers a heavy, steady trickle of bright red blood from the vagina in the presence of a firm fundus. Which potential cause should the nurse question and report to the RN or primary care provider? 1. Uterine atony 2. Laceration 3. Infection of the uterus 4. Perineal hematoma

2. Laceration A steady trickle of blood with a firm uterus is more likely to occur from a laceration rather than from the uterine atony. This type of bleeding is usually bright red in color rather than the dark red color of lochia. A perineal hematoma presents as a bulging, swollen mass on the perineum. Uterine infection typically presents with a foul smelling discharge.

A young client comes to the clinic and requests birth control pills to prevent a pregnancy. She tells the nurse that she is worried that she will get mixed up on what days to take the pills. Which would be the best type of pills to prescribe this client? 1. Oral contraceptive pills packaged with 14 pills 2. Oral contraceptive pills packaged with 28 pills 3. Oral contraceptive pills (OCPs) packaged with 21 pills 4. Oral contraceptive pills packaged with 30 pills

2. Oral contraceptive pills packaged with 28 pills Oral contraceptive pills (OCPs) are packaged with 21 or 28 pills. To eliminate having to count days between pill cycles, most brands are packaged with 28 pills: 21 active pills and 7 placebo pills. There is no need to skip days because the menstrual flow will begin during the 7 days on which she takes the placebo tablets.

An 18-year-old client has given birth at 28 weeks' gestation and her newborn is showing signs of respiratory distress syndrome (RDS). Which statement is true for a newborn with RDS? 1. Respiratory symptoms of RDS typically improve within a short period of time. 2. RDS is caused by a lack of alveolar surfactant. 3. Glucocorticoid (GC) is given to the newborn following birth. 4. RDS is characterized by heart rates below 50 beats per minute.

2. RDS is caused by a lack of alveolar surfactant. Respiratory distress syndrome (RDS) is a serious breathing disorder caused by a lack of alveolar surfactant. Betamethasone, a glucocorticoid, is often given to the mother 12 to 24 hours before a preterm birth to help reduce the severity of RDS, not to the newborn following birth. Respiratory symptoms in the newborn with RDS typically worsen, not improve, within a short period of time after birth. Diagnosis of RDS is made based on a chest X-ray and the clinical symptoms of increasing respiratory distress, crackles, generalized cyanosis, and heart rates exceeding 150 beats per minute (not below 50 beats per minute).

he nurse is teaching a client about mastitis. Which statement should the nurse include in her teaching? 1. A breast abscess is a common complication of mastitis. 2. Symptoms include fever, chills, malaise, and localized breast tenderness. 3. Mastitis usually develops in both breasts of a breastfeeding client. 4. The most common pathogen is group A streptococcus (GAS).

2. Symptoms include fever, chills, malaise, and localized breast tenderness. Mastitis is an infection of the breast characterized by flu-like symptoms, along with redness and tenderness in the breast. The most common causative agent is Staphylococcus aureus. Breast abscess is rarely a complication of mastitis if the client continues to empty the affected breast. Mastitis usually occurs in one breast, not bilaterally.

The nurse is planning an educational session on contraceptives for a group of adolescent high school students. What does the nurse need to do when planning this session? 1. Discuss that the application of a condom should occur after penile-vulvar contact. 2. Teaching about contraceptive options while avoiding indirect encouragement of sexual activity. 3. Explain that oral contraceptive pills (OCPs) are best for adolescents. 4. Argue that encouraging abstinence is unrealistic during the teenage years.

2. Teaching about contraceptive options while avoiding indirect encouragement of sexual activity. The nurse can help the nation achieve the 2020 National Health Goals by teaching adolescents about contraceptive options while being cautious to avoid indirectly encouraging sexual activity among teens. A 2020 National Health Goal is to increase the number of adolescents being instructed on abstinence. A condom should be applied before penile-vulvar contact. Oral contraceptive pills are not the contraceptive of choice for adolescents.

A postpartum client is recovering from the birth and emergent repair of a cervical laceration. Which sign on assessment should the nurse prioritize and report to the health care provider? 1. Warm and flushed skin 2. Weak and rapid pulse 3. Elevated blood pressure 4. Decreased respiratory rate

2. Weak and rapid pulse Excessive hemorrhage puts the client at risk for hypovolemic shock. Signs of impending shock include a weak and rapid pulse, decreased blood pressure, tachypnea, and cool and clammy skin. These findings should be reported immediately to the health care provider so that proper intervention for the client may be instituted.

The neonatal intensive care nurse is assessing a new admission and suspects the newborn to have meconium aspiration syndrome. Which assessment finding would correlate with the nurse's suspicion? 1. PaO2 90 mm/Hg, PaCO2 40 mm/Hg, O2 saturation 96% 2. a barrel-shaped chest with an increased anterior-posterior chest diameter 3. heart rate 110 bpm, respiratory rate 56 breaths/minute, acrocyanosis present 4. a sunken chest with a decreased anterior-posterior chest diameter

2. a barrel-shaped chest with an increased anterior-posterior chest diameter Observe the newborn with MAS for a barrel-shaped chest with an increased anterior-posterior chest diameter, prolonged tachypnea, progression of respiratory distress, intercostal retractions, end-expiratory grunting, and cyanosis. Arterial blood gas analysis will indicate metabolic acidosis with a low blood pH, decreased PaO2, and increased PaCO2. The arterial blood gas values listed are normal as well as the vital signs. Acrocyanosis is a normal expectation of a newborn immediately after birth.

A gravida 5, para 4 is scheduled for a cesarean birth and does not want more children after this one. The client states, "I do not want any hormonal preparations for birth control." Which option is best for this client, and can be performed at the time of the cesarean birth? 1. a vasectomy for the partner 2. a tubal ligation 3. an implanted diaphragm 4. placement of intrauterine device (IUD)

2. a tubal ligation It is best to consider all options for birth control once the decision has been made to not have any further children. Tubal ligation, which can be done easily at the time of a cesarean birth, is a procedure by which the fallopian tubes are occluded by cautery, clamping, crushing, or blocking, thereby preventing the passage of both sperm and ova, preventing further pregnancy. All other options also prevent pregnancy but the tubal ligation is best. A vasectomy may be completed and is a sterilization for the partner. Intrauterine devices (IUDs) and diaphragms require maintenance by the client and are not permanent.

A woman is to receive methotrexate and misoprostol to terminate a first-trimester pregnancy. When preparing the teaching plan for this client, the nurse understands that misoprostol works by: 1. acting as a toxin to the trophoblastic tissue. 2. causing uterine contractions to expel the uterine contents. 3. blocking the action of progesterone on the endometrium. 4. dilating the cervix.

2. causing uterine contractions to expel the uterine contents. Misoprostol works by causing uterine contractions, which help to expel the uterine contents. It has no effect on cervical dilation. Methotrexate is toxic to the trophoblastic tissue. Mifepristone blocks the action of progesterone, which is responsible for preparing the endometrium for implantation and then maintenance of the pregnancy.

The nurse is assessing a newborn suspected of having meconium aspiration syndrome. What sign or symptom would be most suggestive of this condition? 1. intermittent tachypnea 2. expiratory grunting 3. bile-stained emesis 4. high-pitched, shrill cry

2. expiratory grunting Expiratory grunting, a barrel-shaped chest with an increased anterior-posterior chest diameter, prolonged tachypnea, progression from mild-to-severe respiratory distress, intercostal retractions, cyanosis, surfactant dysfunction, airway obstruction, hypoxia, and chemical pneumonitis with inflammation of pulmonary tissues are seen in a newborn with meconium aspiration syndrome. A high-pitched cry may be noted in periventricular hemorrhage/intraventricular hemorrhage. Bile-stained emesis occurs in necrotizing enterocolitis. Intermittent tachypnea can be indicative of transient tachypnea of the newborn or any mild respiratory distress problem.

An obese woman with diabetes has just given birth to a term, large-for-gestational-age (LGA) newborn. Which condition should the nurse most expect to find in this infant? 1. hypertension 2. hypoglycemia 3. hyperglycemia 4. hypotension

2. hypoglycemia LGA infants also need to be carefully assessed for hypoglycemia in the early hours of life because large infants require large amounts of nutritional stores to sustain their weight. If the mother had diabetes that was poorly controlled (the cause of the large size), the infant would have had an increased blood glucose level in utero to match the mother's; this caused the infant to produce elevated levels of insulin. After birth, these increased insulin levels will continue for up to 24 hours of life, possibly causing rebound hypoglycemia.

A nurse is caring for a newborn whose chest X-ray reveals marked hyperaeration mixed with areas of atelectasis. The infant's arterial blood gas analysis indicates metabolic acidosis. For which dangerous condition should the nurse prepare when providing care to this newborn? 1. diaphragmatic hernia 2. meconium aspiration syndrome 3. pneumonia 4. choanal atresia

2. meconium aspiration syndrome The nurse should assess for meconium aspiration syndrome in the newborn. Meconium aspiration involves patchy, fluffy infiltrates unevenly distributed throughout the lungs and marked hyperaeration mixed with areas of atelectasis that can be seen through chest X-rays. Direct visualization of the vocal cords for meconium staining using a laryngoscope can confirm aspiration. Lung auscultation typically reveals coarse crackles and rhonchi. Arterial blood gas analysis will indicate metabolic acidosis with a low blood pH, decreased PaO2, and increased PaCO2. Newborns with choanal atresia, diaphragmatic hernia, and pneumonia do not exhibit these manifestations.

What would the nurse suspect in a premature newborn who has difficulty feeding and maintaining a stable temperature and is vomiting bile? 1. respiratory distress syndrome 2. necrotizing enterocolitis 3. intraventricular hemorrhage 4. meconium aspiration syndrome

2. necrotizing enterocolitis Necrotizing enterocolitis is a disease that is characterized by inflammation in the bowels. It is generally idiopathic and results in difficulty feeding and maintaining thermoregulation, as well as vomiting of bile.

The nurse is meeting with a 36-year-old client who wishes to begin using contraceptives. The client reports being in a long-term, monogamous relationship, runs 2 miles per day, and smokes a pack of cigarettes each day. Which method will the nurse be least likely to suggest to the client? 1. condoms 2. oral contraceptive pills (OCPs) 3. coitus interruptus 4. spermicides

2. oral contraceptive pills (OCPs) Oral contraceptive pills (OCPs) are contraindicated for women who smoke; these women would be at a higher risk for blood clots. Condoms, coitus interruptus, and spermicides can be used at no risk.

A client has just given birth at 42 weeks' gestation. What would the nurse expect to find during her assessment of the neonate? 1. vernix caseosa covering the neonate's body 2. peeling and wrinkling of the neonate's epidermis 3. lanugo covering the neonate's body 4. a sleepy, lethargic neonate

2. peeling and wrinkling of the neonate's epidermis Postdate neonates lose the vernix caseosa, and the epidermis may become peeled and wrinkled. A neonate at 42 weeks' gestation is usually very alert and missing lanugo.

The newborn nursery nurse is admitting a small-for-gestational-age (SGA) infant and is reviewing the maternal history. What factor in the maternal history would the nurse correlate as a risk factor for an SGA infant? 1. grand multiparity 2. placental factors 3. age of 30 years 4. blood group incompatibility

2. placental factors Assessment of the SGA infant begins by reviewing the maternal history to identify risk factors such as placental factors with abnormal umbilical cord insertion, chronic placental abruption (abruptio placentae), malformed and smaller placentas, with placenta previa or placental insufficiency being the main placental causes. Blood group incompatibility, having many pregnancies, and being over the age of 30 will not cause an SGA infant.

A couple has chosen fertility awareness as their method of contraception. The nurse explains that the unsafe period for them during the menstrual cycle would be at which time? 1. six days before the onset of menstruation 2. three days before and three days after ovulation 3. five days after the first day of the menstrual cycle 4. midway between the normal menstrual cycle

2. three days before and three days after ovulation Typically, the unsafe period during the menstrual cycle is approximately 3 days before and 3 days after ovulation. An ovum is released from the ovary 14 days before the next menstrual period.

A nurse is instructing a client on birth control methods. The client asks about the cervical mucus method. When should the nurse tell the client she is fertile in relation to her mucus? 1. when it is thick 2. when it is thin, watery, and copious 3. when it does not stretch 4. Cervical mucus is not a reliable indicator.

2. when it is thin, watery, and copious The cervical mucus method relies on the changes that occur naturally with ovulation. Before ovulation, cervical mucus is thick and does not stretch when pulled. With ovulation, the mucus becomes thin, copious, watery, transparent, and stretchy.

The nurse instructs a client on the use of a vaginal estrogen/progestin (contraceptive) ring. Which client statement indicates that additional instruction is needed? 1. "I will leave the ring in place for 3 weeks." 2. "I leave the ring in place during intercourse." 3. "I am to take the ring out overnight." 4. "I am to use other birth control if I take the ring out for 4 hours."

3. "I am to take the ring out overnight." If the ring is removed for 4 hours for any purpose, it should be replaced with a new ring and a form of barrier protection is to be used for the next 7 days. The ring is not removed overnight. The ring is left in place for 3 weeks and then removed for menstruation during the ring-free week. The ring does not need to be removed for intercourse.

The nurse is caring for a newborn client newly diagnosed with developmental dysplasia of the hip (DDH). Which response by the nurse educates the parents on the correct plan of treatment for this diagnosis? 1. "Treatment will consist of surgery when your child weighs about 10 pounds." 2. "Treatment will start once your child can bear weight." 3. "Treatment will begin immediately." 4. "Treatment will include bilateral casts at 1 month of age."

3. "Treatment will begin immediately." Developmental dysplasia of the hip (DDH) is a congenital newborn condition that requires immediate intervention. The development of the acetabulum of the hip is defective, and it may or may not be dislocated. Treatment of the defect and dislocated hips involves positioning the hip into a flexed, abducted (externally rotated) position to attempt to press the femur into the acetabulum. This involves splints and halters as the first line of treatment. Treatment should not be delayed. Surgery and casts are typically not used as the first line of treatment.

Which assessment by the nurse will best monitor the nutrition and fluid balance in the postterm newborn? Monitor for fall in temperature, indicative of dehydration. Assess for increased muscle tone. Assess for decrease in urinary output. Measure weight once every 2 to 3 days.

3. Assess for decrease in urinary output. The nurse should assess for a decrease in urinary output and fluid balance in the preterm or postterm newborn. Weight of the newborn should be measured daily, not once every 2 to 3 days. Increased muscle tone does not indicate nutrition and fluid imbalance. A rise, not fall, in temperature indicates dehydration.

A newly married couple is meeting with the nurse to discuss a temporary method of birth control that is both a natural form and does not employ birth control pills/devices, in keeping with their religious beliefs. Which method should the nurse point out will best meet their request to delay conception until they are ready? 1. vasectomy 2. coitus interruptus 3. CycleBeads 4. lactation amenorrhea method

3. CycleBeads CycleBeads is a natural fertility awareness method in which a woman uses a special circle of beads that helps her predict fertile days so that she can abstain from sex on those days. This method meets all of the requirements of the couple. Lactation amenorrhea is a natural method, but it would not apply in this situation, as the woman is not currently breastfeeding. Coitus interruptus is also a natural method, but it is not a fertility awareness method. Vasectomy is a surgical procedure that results in permanent sterility. As the couple would like to have children in the future, this method would not be appropriate.

A nurse is caring for a preterm newborn born at 29 weeks' gestation. Which nursing diagnosis would have the highest priority? 1. Grieving related to the loss of "a healthy full-term newborn" 2. Risk for injury related to the very thin epidermis layer of skin 3. Ineffective thermoregulation related to decreased amount of subcutaneous fat 4. Imbalanced nutrition: Less than body requirements related to the premature digestive system

3. Ineffective thermoregulation related to decreased amount of subcutaneous fat In the condition of hypothermia, newborns typically metabolize brown fat. This requires the newborn to use glucose and oxygen. A premature infant is at risk for respiratory distress and hypoglycemia. The hypoglycemia can increase the infant's need for glucose and oxygen, which, in turn, could cause more severe disease or further complications. The other diagnoses are appropriate but not the highest priority.

An infant is suffering from neonatal abstinence syndrome. The nurse provides appropriate care and support for the infant during the infant's time on the unit. Besides nursing and medical care, what other step would the nurse take to support the infant? 1. Make sure the infant was in a bright, loud room. 2. Contact the chaplain. 3. Link the family with community sources for aid. 4. Make sure a volunteer feeds the baby.

3. Link the family with community sources for aid. Besides nursing care, the nurse would make sure that interdisciplinary members of the team were involved, such as the doctor, nutritionist, and community worker, and provide a supportive environment for the family and the client. The chaplain may not be support for infant or family may not be present. The bright room is overly stimulating to the withdrawing infant, and feeding is basic care.

The nurse is admitting a newborn into the nursery and notes the newborn was born at 42 weeks' gestation. Which action should the nurse prioritize when caring for this infant? 1. Anticipate a blood transfusion will be required 2. Wrap the newborn lightly in a single blanket. 3. Monitor the newborn's blood glucose levels. 4. Ensure feedings are offered every 4 hours.

3. Monitor the newborn's blood glucose levels. The infant is a post-term infant because of the gestational age. In the last weeks of gestation, the infant relies on glycogen for nutrition. This depletes the liver glycogen stores and may lead to neonatal hypoglycemia, which will require monitoring of serial blood glucose levels. The newborn may require intravenous glucose infusions to stabilize the glucose level. Post-term newborns can have thermoregulation concerns, so a thermoneutral environment such as a radiant heat warmer or isolette may be needed. Use measures to minimize heat loss. Post-term newborns are ravenous eaters at birth and may be offered feedings at 1 or 2 hours of age and as desired after that. Although exchange transfusion may be done to prevent or treat hyperviscosity, this is not a common concern so the glucose levels are a higher priority.

A nurse is planning care for a client and her husband recently diagnosed with multiple sclerosis and wanting to prevent pregnancy for now. What is the most appropriate nursing diagnosis for this couple? 1. Decisional conflict regarding choice of birth control because of health concerns 2. Altered sexuality pattern related to fear of pregnancy 3. Readiness for enhanced knowledge regarding contraception options 4. Risk for ineffective health maintenance related to lack of knowledge

3. Readiness for enhanced knowledge regarding contraception options The nurse should recognize that the couple is ready for enhanced knowledge regarding contraceptive options. By providing education materials on the options available the clients can select the best option for them.

The nursery nurse notes that one of the newborn infants has white patches on his tongue that look like milk curds. What action would be appropriate for the nurse to take? 1. Since it looks like a milk curd, no action is needed. 2. Rinse the tongue off with sterile water and a cotton swab. 3. Report the finding to the pediatrician. 4. Wipe the tongue off vigorously to remove the white patches.

3. Report the finding to the pediatrician. Although the finding looks like a milk curd, if the white patch remains after feeding, the pediatrician needs to be notified. The likely cause of the white patch on the tongue is a fungal infection called Candida albicans, which the newborn probably contracted while passing through the birth canal. The nurse should not try to remove the patches.

A client with liver disease is seen in the clinic wanting to begin contraception. The nurse recognizes that which type of contraception is best for this client? 1. Oral contraceptive pills (OCPs) 2. Intramuscular injections 3. Vaginal estrogen/progestin (contraceptive) ring 4. Subdermal hormone implant

3. Vaginal estrogen/progestin (contraceptive) ring The nurse should recognize that the vaginal estrogen/progestin ring releases hormones into the mucous membranes of the vagina and avoids the "first pass" through the liver.

What postpartum client should the nurse monitor most closely for signs of a postpartum infection? 1. a primiparous client who had a vaginal birth 2. a client who had an 8-hour labor 3. a client who had a nonelective cesarean birth 4. a client who conceived following fertility treatments

3. a client who had a nonelective cesarean birth The major risk factor for postpartum infection is a nonelective cesarean birth. Antepartum risk factors include history of infection; history of chronic conditions, such as diabetes, anemia, or poor nutrition; infections of the genital tract; smoking; and obesity. The other listed factors are not noted risk factors for infection.

What is a typical feature of a small-for-gestational-age (SGA) newborn that differentiates it from a preterm baby with a low-birth-weight? 1. decreased body temperature 2. decreased muscle mass 3. ability to tolerate early oral feeding 4. face is angular and pinched

3. ability to tolerate early oral feeding Unlike preterm babies with low birth weights, a small-for-gestational-age baby can safely tolerate early oral feeding. It usually has a coordinated sucking and swallowing reflex. Decreased muscle mass, decreased body temperature, and an angular and pinched face are features common to both an SGA baby and a preterm baby.

A client has given birth to a small-for-gestational-age (SGA) newborn. Which finding would the nurse expect to assess? 1. brown lanugo body hair 2. round flushed face 3. head larger than body 4. protuberant abdomen

3. head larger than body A small-for-gestational-age (SGA) newborn will typically have a head that is larger than the rest of his or her body. SGA newborns weigh below the 10th percentile on the intrauterine growth chart for gestational age. They have an angular and pinched face and not a rounded and flushed face. Round flushed face and protuberant abdomen are the characteristic features of large-for-gestational-age (LGA) newborns. Preterm newborns, and not SGA newborns, are covered with brown lanugo hair all over the body.

An expectant mother is on heparin for previous blood clots and voicing concerns about how her medications will affect her baby. The nurse would inform the mother that: 1. it is recommended to stop taking the heparin while she is pregnant. 2. she should discontinue the heparin and change to another anticoagulant. 3. heparin does not cross the placenta and is safe for her to take. 4. any medication that an expectant mother takes can cause sequelae for the infant.

3. heparin does not cross the placenta and is safe for her to take Heparin is a medication that does not cross the placenta and therefore is safe to use during pregnancy. Not all medications cause fetal sequelae. It is not recommended to abruptly discontinue any medication without consulting the mother's health care provider. Heparin is the safest anticoagulant for a pregnant woman to take.

The nurse is assessing a client 48 hours postpartum and notes on assessment: temperature 101.2oF (38.4oC), HR 82, RR 18, BP 125/78 mm Hg. The nurse should suspect the vital signs indicate which potential situation? 1. Shock 2. Normal vital signs 3. Infection 4. Dehydration

3. infection Temperatures elevated above 100.4° F (38° C) 24 hours after birth are indicative of possible infection. All but the temperature for this client are within normal limits, so they are not indicative of shock or dehydration.

A client in her seventh week of the postpartum period is experiencing bouts of sadness and insomnia. The nurse suspects that the client may have developed postpartum depression. What signs or symptoms are indicative of postpartum depression? Select all that apply. 1. manifestations of mania 2. bizarre behavior 3. loss of confidence 4. decreased interest in life 5. inability to concentrate

3. loss of confidence 4. decreased interest in life 5. inability to concentrate The nurse should monitor the client for symptoms such as inability to concentrate, loss of confidence, and decreased interest in life to verify the presence of postpartum depression. Manifestations of mania and bizarre behavior are noted in clients with postpartum psychosis.

The nurse is providing care to a newborn with severe meconium aspiration syndrome (MAS). The nurse is reviewing the newborn's diagnostic test results. Which finding would the nurse expect? 1. elevated blood pH 2. increased PaO2 3. patchy, fluffy infiltrates on chest X-ray 4. vocal cords negative for meconium

3. patchy, fluffy infiltrates on chest X-ray Chest X-rays show patchy, fluffy infiltrates unevenly distributed throughout the lungs and marked hyperaeration mixed with areas of atelectasis. Arterial blood gases analysis will indicate metabolic acidosis with a low blood pH, decreased PaO2, and increased PaCO2. Direct visualization of the vocal cords for meconium staining using an appropriate size laryngoscope is needed.

Every postpartum client has the potential of hemorrhage. While assessing a client's status, which finding would be of little benefit in identifying the possibility of hemorrhage? 1. vital signs 2. uterine tone 3. signs of shock 4. estimated amount of blood loss

3. signs of shock Signs of shock do not appear until the hemorrhage is far advanced due to the increased fluid and blood volume of pregnancy. Vital signs would show an increased pulse rate and decreased level of consciousness. The amount of lochia would be much greater than usual, and urinary output would be diminished with signs of acute renal failure. The uterus may also appear soft and spongy instead of firm.

Depot medroxyprogesterone acetate (Depo-Provera) has been suggested to a postpartum client as a method of birth control. Which statement by the client reveals to the nurse an accurate understanding and benefit to using this type of birth control? 1. "I like that this is only a yearly injection." 2. "Weight loss of 5 to 10 lb (2.3 to 4.6 kg) is expected." 3. "My acne should diminish within 30 days of birth control use." 4. "I am able to breastfeed while using this type of birth control."

4. "I am able to breastfeed while using this type of birth control." Depo-Provera is a progestin-only hormonal birth control system. The ability to breastfeed while using it is a benefit to postpartum clients who desire to breastfeed. One injection can prevent pregnancy for 12 weeks and is 99.7% effective. Intramuscular injections are scheduled for every 3 months and can begin approximately 6 weeks following birth. Weight gain, not weight loss, may occur. Irregular bleeding is the most common side effect. Acne, abdominal discomfort, and osteoporosis are also adverse effects.

A couple comes to the clinic for a fertility evaluation. The male partner is to undergo a semen analysis. After teaching the partner about this test, which client statement indicates that the client has understood the instructions? 1. "I will place the specimen in a special plastic bag to transport it." 2. "I will withdraw before I ejaculate during sex to collect the specimen." 3. "I need to bring the specimen to the lab the day after collecting it." 4. "I have to abstain from sexual activity for about 2 to 5 days before the sample."

4. "I have to abstain from sexual activity for about 2 to 5 days before the sample." Semen analysis is the most important indicator of male fertility. The man should abstain from sexual activity for 2 to 5 days before giving the sample. For a semen examination, the man is asked to produce a specimen by ejaculating into a specimen container and delivering it to the laboratory for analysis within 1 hour. When the specimen is brought to the laboratory, it is analyzed for volume, viscosity, number of sperm, sperm viability, motility, and sperm shape.

A woman is going to have in vitro fertilization. When preparing her for this, the nurse would make which statement? 1. "Most procedures are effective the first time tried." 2. "You will need to select a surrogate mother." 3. "This is dangerous if there is ovarian cancer in your family." 4. "It can be done with frozen donor sperm."

4. "It can be done with frozen donor sperm." Fresh or frozen sperm may be used. The success of in vitro fertilization is not related to the incidence of ovarian cancer in the family. Quite often, more than one attempt is needed before successful implantation.

A premature infant is admitted to the neonatal intensive care unit with respiratory distress syndrome and requires assisted ventilation. The parents asks the nurse, "Why won't our baby breath on its own?" What is the nurse's best response? 1. "Premature infants have a respiratory system that takes time to adjust to extrauterine life." 2. "Most preterm infants require additional oxygen through ventilation to sustain respiration." 3. "Your infant's cardiovascular system is not developed yet in order to sustain respiration." 4. "Your infant cannot sustain respirations yet due to the lack of assistance from surfactant."

4. "Your infant cannot sustain respirations yet due to the lack of assistance from surfactant." Preterm infants lacks surfactant to lower the surface tension in the alveoli and stabilize them to prevent their collapse. Even if preterm newborns can initiate respirations, they have a limited ability to retain air due to insufficient surfactant. Preterm newborns develop atelectasis quickly without alveoli stabilization. Fetal circulation patterns persist.

A nurse is caring for a newborn client diagnosed with spina bifida. Which assessment finding would be a priority for the nurse who is monitoring for the risk of hydrocephalus? 1. Assess the motor function of the lower extremities. 2. Assess the newborn's neurological response. 3. Assess the newborn's weight. 4. Assess head circumference measurements.

4. Assess head circumference measurements. Hydrocephalus is the extra accumulation of CSF fluid in the ventricles of the brain, which causes dilation. One of the main symptoms of hydrocephalus is increasing measurements of head circumference. A change in neurological status in hydrocephalus may not occur immediately but may be assessed at a later point. Weighing a newborn and checking motor function will not be a significant indicator of hydrocephalus.

A client is requesting information on the various available contraceptives. When explaining a vaginal spermicide, which information should the nurse prioritize? 1. Wash with clean water and soap after intercourse. 2. Apply the spermicide directly to the glans penis to kill the sperm when they exit the penis. 3. Leave the product in place for 24 hours after intercourse. 4. Insert the product by applicator in the vagina prior to intercourse.

4. Insert the product by applicator in the vagina prior to intercourse. Vaginal spermicides provide a physical barrier that prevents sperm penetration and a chemical barrier that kills sperm. It is designed to be inserted vaginally immediately before or within a few hours before sexual intercourse. Hygiene is always important and is unrelated to the use of spermicides. Applying the spermicide to the penis would be ineffective as it would rub off. The product is immediately effective and can be removed right after intercourse.

Which intervention is helpful for the neonate experiencing drug withdrawal? 1. Dress the neonate in loose clothing so the infant will not feel restricted. 2. Place the isolette near the nurses' station for frequent contact with health care workers. 3. Withhold all medication to help the liver metabolize drugs. 4. Place the isolette in a quiet area of the nursery.

4. Place the isolette in a quiet area of the nursery. Neonates experiencing drug withdrawal commonly have sleep disturbance. The neonate should be moved to a quiet area of the nursery to minimize environmental stimuli. Medications such as phenobarbital and paregoric should be given as needed. The neonate should be swaddled to prevent him from flailing and stimulating himself.

The father of a 2-week-old infant presents to the clinic with his disheveled wife for a postpartum visit. He reports his wife is acting differently, is extremely talkative and energetic, sleeping only 1 or 2 hours at a time (if at all), not eating, and appears to be totally neglecting the infant. The nurse should suspect the client is exhibiting signs and symptoms of which disorder? 1. Postpartum blues 2. Maladjustment 3. Postpartum psychosis 4. Postpartum depression

4. Postpartum psychosis Postpartum psychosis in a client can present with extreme mood changes and odd behavior. Her sudden change in behavior from normal, along with a lack of self-care and care for the infant, are signs of psychosis and need to be assessed by a provider as soon as possible. Postpartum depression affects the woman's ability to function; however, her perception of reality remains intact. Postpartum blues is a transitory phase of sadness and crying common among postpartum women.

The nurse is conducting a class for pregnant women on problems associated with infections seen in women during gestation. One mother tells the nurse that she has never had chickenpox (varicella) and is worried she will contract it before she delivers. What would the nurse explain to this mother to ease her anxiety? 1. The likelihood of her catching varicella is very slim, so tell her not to worry. 2. Even if she does get varicella, her baby will be fine since varicella is not passed on to the fetus. 3. She needs to let her physician know that she has not had varicella so she can get immunized at her next visit. 4. She can receive her varicella vaccine immediately after delivery, followed by a second dose at her 6-week postpartum visit.

4. She can receive her varicella vaccine immediately after delivery, followed by a second dose at her 6-week postpartum visit. Varicella zoster is spread by respiratory droplets and is easily contracted if the mother is exposed. If the mother does contract varicella, especially from weeks 12 to 20 of gestation, it can be devastating to the fetus. Birth defects, central nervous system damage, low birth weight, cognitive delays and deafness can all occur. However, a mother cannot receive a varicella immunization during pregnancy since the immunization can cross the placenta and act like the actual disease, causing the same effects. The mother is immunized prior to discharge after delivery and at her 6-week postpartum visit.

The client brings her infant daughter to the pediatrician's office for her first visit since hospital discharge. At birth, the newborn was at the 8th percentile with a weight of 2,350 g. She was born at 36 weeks' gestation. Which documentation is most accurate? 1. The infant was born at term but a very-low-birth-weight and small-for-gestational-age. 2. The infant was born at term but at a low-birth-weight and small-for-gestational- age. 3. The infant was a preterm, very-low-birth-weight and small-for-gestational-age. 4. The infant was a preterm, low-birth-weight and small-for-gestational-age neonate.

4. The infant was a preterm, low-birth-weight and small-for-gestational-age neonate. Born at 36 weeks' gestation is a preterm age (under 37 weeks). The infant was a low-birth-weight (under 2,500 g) and small-for-gestational-age infant at the 8th percentile (under the 10th percentile). The other documentations are not accurate.

The nurse is teaching a young couple who desire to start their family the various methods for determining fertility. After discovering the woman regularly travels internationally for work, deals with a lot of job anxiety, and frequently uses an electric blanket at home, the nurse will discourage the use of which method? 1. cervical mucus method 2. symptothermal method 3. calendar method 4. basal body temperature method

4. basal body temperature method BBT is a method where the body temperature should be checked and recorded first thing in the morning, immediately after waking and before getting out of bed. It is important for the client to maintain a normal bedtime routine. Use of an electric blanket, stress, and anxiety can cause a false elevation in the BBT. The calendar method would depend upon her schedule. Cervical mucus and symptothermal methods would be viable options.

Which measurement best describes delayed postpartum hemorrhage? 1. blood loss in excess of 3000 ml, occurring at least 24 hours and up to 12 weeks after birth 2. blood loss in excess of 300 ml, occurring within the first 24 hours after birth 3. blood loss in excess of 1,000 ml, occurring within 24 hours after birth 4. blood loss in excess of 500 ml, occurring at least 24 hours and up to 12 weeks after birth

4. blood loss in excess of 500 ml, occurring at least 24 hours and up to 12 weeks after birth Late postpartum hemorrhage involves blood loss in excess of 500 ml. Most delayed postpartum hemorrhages occur between the fourth and ninth days postpartum, but can occur any time between 24 hours and 12 weeks after birth The most common causes of a delayed postpartum hemorrhage include retained placental fragments, intrauterine infection, and fibroids.

A young couple is exploring their contraceptive options and are curious about using an intrauterine contraceptive device. The nurse explains that there are two types, one that uses hormones and one that uses: 1. magnesium. 2. silicone. 3. potassium. 4. copper.

4. copper The implants contain either copper or progesterone to enhance their effectiveness. One or two attached strings protrude into the vagina so that the user can check its placement. Four IUCs are currently available in the United States: the copper ParaGard-TCu-380A, the levonorgestrel-releasing intrauterine systems (LNG-IUSs) marketed as Mirena and Kyleena, and the LNG-IUD marketed as Jaydess. The ParaGard-TCu-380A is approved for 10 years of use and is nonhormonal. Its mechanism of action is based on the release of copper ions, which alone are spermicidal. Additionally, the device causes an inflammatory action leading to a hostile uterine environment.

What is a consequence of hypothermia in a newborn? 1. respirations of 46 2. skin pink and warm 3. heart rate of 126 4. holds breath 25 seconds

4. holds breath 25 seconds Apnea is the cessation of breathing for a specific amount of time, and in newborns it usually occurs when the breath is held for 15 seconds. Apnea, cyanosis, respiratory distress, and increased oxygen demand are all consequences of hypothermia.

A newborn is designated as very-low-birth-weight. When weighing this newborn, the nurse would expect to find which weight? 1. approximately 2,500 g 2. more than 4,000 g 3. less than 1,000 g 4. less than 1,500 g

4. less than 1,500 g A very-low-birth-weight newborn weighs less than 1,500 g. A large-for-gestational-age newborn typically weighs more than 4,000 g. A small-for-gestational-age newborn or a low-birth-weight newborn typically weighs about 2,500 g. An extremely-low-birth-weight newborn weighs less than 1,000 g.

Which symptom would most accurately indicate that a newborn has experienced meconium aspiration during the birth process? 1. bluish skin discoloration 2. listlessness or lethargy 3. stained umbilical cord and skin 4. meconium stained fluids followed by tachypnea

4. meconium stained fluids followed by tachypnea Meconium stained cord and skin indicates a potential of meconium aspiration, and the nurse should inform the primary care provider. But if the infant actually experiences respiratory distress following a birth with meconium stained fluids, the likelihood of meconium aspiration is greatly increased. Listlessness or lethargy by themselves does not indicate meconium aspiration. Bluish skin discoloration is normal in infants shortly after birth until the infant's respiratory system clears out all the amniotic fluid.

The nurse assesses a post-term newborn. What finding corresponds with this gestational age diagnosis? 1. abundant vernix caseosa and lanugo 2. Wharton's jelly 3. few creases on soles 4. meconium-stained skin and fingernails

4. meconium-stained skin and fingernails Postterm newborns typically exhibit the following characteristics: dry, cracked, peeling, wrinkled skin; vernix caseosa and lanugo are absent; long, thin extremities; creases that cover the entire soles of the feet; abundant hair on scalp; thin umbilical cord; long fingernails; limited vernix and lanugo; and meconium-stained skin and fingernails.

The nurse notes the listed assessment findings in a newborn. Which finding correlates with a glucose level for a newborn is 39 mg/dl (2.16 mmol/l). 1. strong cry 2. elevated temperature 3. heart rate of 142 beats/min 4. poor feeding

4. poor feeding The nurse will further assess the newborn for signs of poor feeding, which is an early sign of low glucose. Normal newborn glucose levels range from 45 mg/dl (2.5 mmol/l) to 126 mg/dl (7.0 mmol/l). The newborn's heart rate is normal. Normal newborn heart rate ranges from 100 beats/min to 205 beats/min while awake. The temperature may be decreased, not elevated.

A newborn girl who was born at 38 weeks' gestation weighs 2000 g and is below the 10th percentile in weight. The nurse recognizes that this girl will be placed in which classification? 1. term, small-for-gestational-age, and very-low-birth-weight infant 2. late preterm and appropriate for gestational age 3. late preterm, large-for-gestational-age, and low-birth-weight infant 4. term, small-for-gestational-age, and low-birth-weight infant

4. term, small-for-gestational-age, and low-birth-weight infant Infants born before term (before the beginning of the 38th week of pregnancy) are classified as preterm infants, regardless of their birth weight. Term infants are those born after the beginning of week 38 and before week 42 of pregnancy. Infants who fall between the 10th and 90th percentiles of weight for their gestational age, whether they are preterm, term, or postterm, are considered appropriate for gestational age (AGA). Infants who fall below the 10th percentile of weight for their age are considered small-for-gestational-age (SGA). Those who fall above the 90th percentile in weight are considered large-for-gestational-age (LGA). Still another term used is low-birth-weight (LBW; one weighing under 2500 g at birth). Those weighing 1000 to 1500 g are very-low-birth-weight (VLBW). Those born weighing 500 to 1000 g are considered extremely-low-birth-weight infants (ELBW).

A client who gave birth several hours ago is experiencing postpartum hemorrhage. She had a cesarean birth and received deep, general anesthesia. She has a history of postpartum hemorrhage with her previous births. The blood is a dark red. Which cause of the hemorrhage is most likely in this client? 1. cervical laceration 2. retained placental fragment 3. disseminated intravascular coagulation 4. uterine atony

4. uterine atony Uterine atony, or relaxation of the uterus, is the most frequent cause of postpartum hemorrhage; it tends to occur most often in Asian or Hispanic woman. Conditions that contribute to uterine atony include having received deep anesthesia or analgesia and a prior history of postpartum hemorrhage. A cervical laceration is less likely because the blood is dark, not bright red, and bleeding from such a laceration usually occurs immediately after detachment of the placenta. Disseminated intravascular coagulation is typically associated with premature separation of the placenta, a missed early miscarriage, or fetal death, none of which is evident in this scenario. A retained placental fragment is possible, and could contribute to the atony, but there is no evidence for this in the scenario.

A nursing instructor is teaching about changes the newborn must make to survive outside of the uterus. The instructor realizes that further teaching is needed when a student makes which statement? "The baby takes the first breath when ready to leave the uterus." "The baby takes the first breath when stimulated by a slight slap." "The baby's lungs begin to function when the umbilical cord is clamped." "The baby takes the first breath when the umbilical cord is clamped."

1. "The baby takes the first breath when ready to leave the uterus." When the baby's umbilical cord is clamped, the baby takes the first breath and the lungs begin to function. The breath usually occurs when the baby is stimulated by a slight slap. The baby takes the first breath within 10 seconds post birth, not when ready to leave the uterus.

The nurse is weighing an infant and is ensuring that the scale is warmed and the procedure is performed as quickly as possible. Doing so allows the nurse to minimize the effects of heat loss by what method?

1. Conduction Conductive heat loss occurs when the newborn's skin touches a cold surface, causing body heat to transfer to the colder object. An example of this is when the infant is placed on a cold scale. Heat loss by convection happens when air currents blow over the newborn's body. An example of this is when the infant is left in a draft of cool air. Evaporative heat loss happens when the newborn's skin is wet. Heat loss also occurs by radiation to a cold object that is close to but not touching the newborn.

When assessing a 1-day postpartum woman's pulse, what is the first action a nurse should take in response to a rate of 56 beats/min? 1. Do nothing, this is normal. 2. Compare the pulse rate with her pulse rate on the first prenatal care visit. 3. Advise that the client not get out of bed until the nurse returns with assistance. 4. Ask the client what she has had to eat today.

1. Do nothing, this is normal. During pregnancy, the distended uterus obstructs the amount of venous blood returning to the heart; after birth, to accommodate the increased blood volume returning to the heart, stroke volume increases. Increased stroke volume reduces the pulse rate to between 50 and 70 beats per minute. The nurse should be certain to compare a woman's pulse rate with the slower range expected in the postpartum period, not with the normal pulse rate in the general population. Pulse usually stabilizes to prepregnancy levels within 10 days.

When teaching the new mother about breastfeeding, the nurse is correct when providing what instructions? Select all that apply. 1. Encourage breastfeeding of the newborn infant on demand. 2. Place baby in uninterrupted skin-to-skin contact (kangaroo care) with the mother. 3. Provide breastfeeding newborns with pacifiers. 4. Help the mother initiate breastfeeding within 30 minutes of birth. 5. Give newborns water and other foods to balance nutritional needs.

1. Encourage breastfeeding of the newborn infant on demand. 2. Place baby in uninterrupted skin-to-skin contact (kangaroo care) with the mother. 4. Help the mother initiate breastfeeding within 30 minutes of birth.

The nurse is conducting a postpartum examination on a client who reports pain and is unable to sit comfortably. The perineal exam reveals an episiotomy without signs of a hematoma. Which action should the nurse prioritize? 1. Place an ice pack. 2. Apply a warm washcloth. 3. Put on a witch hazel pad. 4. Notify a health care provider.

1. Place an ice pack. The labia and perineum may be bruised and edematous after birth; the use of ice would assist in decreasing the pain and swelling. Applying a warm washcloth would bring more blood as well as fluid to the sore area, thereby increasing the edema and the soreness. Applying a witch hazel pad needs the order of the health care provider. Notifying a health care provider is not necessary at this time as this is considered a normal finding.

A nurse is required to obtain the temperature of a healthy newborn who was placed in an open crib. Which is the most appropriate method for measuring a newborn's temperature? 1. Place electronic temperature probe in the midaxillary area. 2. Obtain the temperature orally. 3. Tape electronic thermistor probe to the abdominal skin. 4. Obtain the temperature rectally.

1. Place electronic temperature probe in the midaxillary area.

A preterm infant will be hospitalized for an extended time. Assuming the infant's condition is improving, which environment would the nurse feel is most suitable for the child? 1. Provide a mobile the child can see no matter how he or she is turned. 2. Keep the environment free of color to reduce eye straining. 3. Bring the child's open bassinet near the desk area so the infant sees people. 4. Place the infant's Isolette near the window so the child can see outside.

1. Provide a mobile the child can see no matter how he or she is turned. Preterm infants are able to focus at short distances before they can see well at long distances. A mobile offers short-distance stimulation.

The nurse is looking over a newborn's plan of care regarding expected outcomes. Which outcome would not be appropriate according to a newborn's nursing care? 1. The newborn will experience no bleeding episodes lasting more than 5 minutes. 2. The newborn's blood glucose will remain above 50 mg/dl 3. The newborn will be correctly identified prior to separation from the parents. 4. The newborn's body temperature will stabilize between 97.8ºF and 99.5ºF (36.6ºC and 37.5ºC).

1. The newborn will experience no bleeding episodes lasting more than 5 minutes. Bleeding episodes should not be occurring at all, and any episodes should be reported to the physician immediately if not responsive to immediate action to stop it. All other outcomes are pertinent to the newborn's care.

Which statement is true regarding fetal and newborn senses? 1. The rooting reflex is an example that the newborn has a sense of touch. 2. A newborn cannot experience pain. 3. A newborn does not have the ability to discriminate between tastes. 4. A fetus is unable to hear in utero. 5. A newborn cannot see until several hours after birth.

1. The rooting reflex is an example that the newborn has a sense of touch. The rooting reflex is an example of a newborn's sense of touch. Newborns experience pain, have vision, and can discriminate between tastes. The fetus can hear in utero.

The nurse is preparing new parents and their infant for discharge by answering questions and presenting basic discharge instruction. Which explanation should the nurse provide when questioned about the infant's yellow hue? 1. The tint is due to jaundice. 2. It's a mild reaction to the vitamin K injection. 3. The infant needs to be in the sunlight to clear the skin. 4. Yellow is the normal color for some newborns.

1. The tint is due to jaundice. Newborns often have a yellow tint to the skin if the newborn develops jaundice. Any newborn developing jaundice needs to be assessed by the health care provider. Jaundice in the first 24 hours is pathologic and must be reported immediately to the health care provider. Jaundice after 2 days is considered physiologic and is due to the liver's inability to adequately process bilirubin which seeps into the tissues, giving the skin a yellowish color. It is not considered normal and does require assessment and intervention. Phototherapy is the recommended treatment of choice, not putting the child in sunlight. It is not a reaction to the vitamin K injection.

The client, who has just been walking around her room, sits down and reports leg tightness and achiness. After resting, she states she is feeling much better. The nurse recognizes that this discomfort could be due to which cause? 1. thromboembolic disorder of the lower extremities 2. hormonal shifting of relaxin and estrogen 3. infection 4. normal response to the body converting back to prepregnancy state

1. Thromboembolic disorder of the lower extremities Thromboembolic disorders may present with subtle changes that must be evaluated with more than just physical examination. The woman may report lower extremity tightness or aching when ambulating that is relieved with rest and elevation. Edema in the affected leg, along with warmth and tenderness and a low grade fever, may also be noted. The woman's complaints do not reflect a normal hormonal response, infection, or the body converting back to the prepregnancy state.

One thing a new mother does is to adapt to the new baby psychologically. The woman takes on her new role as a mother by going through a series of four developmental stages. What is one of them? 1. achieving a maternal identity 2. finding a way to get the new baby to conform to existing family interrelationships 3. preparing for the infant before she conceives 4. physical restoration and learning to get help in caring for the infant

1. achieving a maternal identity The woman adapts to her new role as a mother through a series of four developmental stages:1. Beginning attachment and preparation for the infant during pregnancy2. Increasing attachment, learning to care for the infant, and physical restoration during the early postpartum period3. Moving toward a new normal in the first several months4. Achieving a maternal identity around 4 months

The nurse institutes measures to maintain thermoregulation based on the understanding that newborns have limited ability to regulate body temperature because they: 1. are unable to shiver effectively to increase heat production. 2. have an abundant amount of subcutaneous fat all over. 3. lose more body heat when they sweat than adults. 4. have a smaller body surface compared to body mass.

1. are unable to shiver effectively to increase heat production. Newborns have difficulty maintaining their body heat through shivering and other mechanisms. They have a large body surface area relative to body weight and have limited sweating ability. Additionally, newborns lack subcutaneous fat to provide insulation.

A nurse finds the uterus of a postpartum woman to be boggy and somewhat relaxed. This a sign of which condition? 1. atony 2. hemorrhage 3. normal involution 4. infection

1. atony The uterus in a postpartum client should be midline and firm. A boggy or relaxed uterus signifies uterine atony, which can predispose the woman to hemorrhage.

When caring for a postpartum client who has given birth vaginally, the nurse assesses the client's respiratory status, noting that it has quickly returned to normal. The nurse understands that which factor is responsible for this change? 1. decreased intra-abdominal pressure 2. decreased bladder pressure 3. increased progesterone levels 4. use of anesthesia during birth

1. decreased intra-abdominal pressure The nurse should identify decreased intra-abdominal pressure as the cause of the respiratory system functioning normally. Progesterone levels do not influence the respiratory system. Decreased bladder pressure does not affect breathing. Anesthesia used during birth causes the respiratory system to take a longer time to return to normal.

The nurse is describing fetal circulation to new parents and how the circulation changes after birth. The nurse describes a structure that allows the pulmonary circulation to be bypassed, but that shortly after birth this structure should close. Which structure is the nurse describing? 1. ductus arteriosus 2. foramen ovale 3. ductus venosus 4. umbilical vessels

1. ductus arteriosus During fetal life, the ductus arteriosus protects the lungs against circulatory overload by shunting blood into the descending aorta, bypassing the pulmonary circulation. The foramen ovale is located in the septum between the atria and allowed blood to flow from the right atrium directly the left atrium. The ductus venous allowed the majority of the blood to bypass the liver. The umbilical vessels carried oxygenated blood to the fetus and removed deoxygenated blood and waste products from the fetus.

A nurse is making a home visit to a postpartum woman who gave birth to a healthy newborn 4 days ago. The woman's breasts are swollen, hard, and tender to the touch. The nurse documents this finding as: 1. engorgement. 2. involution. 3. mastitis. 4. engrossment.

1. engorgement. Engorgement is the process of swelling of the breast tissue as a result of an increase in blood and lymph supply as a precursor to lactation (Figure 15.4). Breast engorgement usually peaks in 3 to 5 days postpartum and usually subsides within the next 24 to 36 hours (Chapman, 2011). Engorgement can occur from infrequent feeding or ineffective emptying of the breasts and typically lasts about 24 hours. Breasts increase in vascularity and swell in response to prolactin 2 to 4 days after birth. If engorged, the breasts will be hard and tender to touch. Involution refers to the process of the uterus returning to its prepregnant state. Mastitis refers to an infection of the breasts. Engrossment refers to the bond that develops between the father and the newborn.

A client experienced prolonged labor with prolonged premature rupture of membranes. The nurse would be alert for which condition in the mother and the newborn? 1. infection 2. hemorrhage 3. trauma 4. hypovolemia

1. infection Although hemorrhage, trauma, and hypovolemia may be problems, the prolonged labor with the prolonged premature rupture of membranes places the client at high risk for a postpartum infection. The rupture of membranes removes the barrier of amniotic fluid, so bacteria can ascend.

During an assessment, the nurse suspects a newborn has a chromosomal disorder. What did the nurse most likely assess in the baby? 1. low-set ears 2. bowed legs 3. short neck 4. slanting of the palpebral fissure

1. low-set ears Ears that are low-set ears is a common assessment finding in newborns with the trisomy chromosome abnormalities. Short neck, bowed legs, and slanting of the palpebral fissure are less common findings in a newborn with a chromosomal disorder.

Which condition should the nurse look for in the client's history that may explain an increase in the severity of afterpains? 1. multiple gestation 2. diabetes 3. bottle-feeding 4. primiparity

1. multiple gestation Multiple gestation, breastfeeding, multiparity, and conditions that cause overdistention of the uterus will increase the intensity of afterpains. Bottle-feeding and diabetes aren't directly associated with increasing severity of afterpains, unless the client has delivered a macrosomic neonate.

A nurse is reviewing the laboratory test results of a newborn. Which result would the nurse identify as a cause for concern? 1. platelets 75,000/µL 2. hemoglobin 19 g/dL (190 g/L) 3. white blood cells 20,000/mm3 4. hematocrit 52% (0.52)

1. platelets 75,000/µL Normal newborn platelets range from 150,00 to 350,000/µL. Normal hemoglobin ranges from 17 to 23 g/dL (170 to 230 g/L), and normal hematocrit ranges from 46% to 68%. Normal white blood cell count ranges from 10,000 to 30,000/mm3.

The birth center recognizes that attachment is very important in the early stages after birth. Which policy would be inappropriate for the birth center to implement when assisting new parents in this process? 1. policies that discourage unwrapping and exploring the infant 2. policies that allow flexibility for cultural differences 3. policies that allow visitors 4. policies that allow rooming the infant and mother together

1. policies that discourage unwrapping and exploring the infant Various factors associated with the health care facility or birthing unit can hinder attachment. These may include separation of infant and parents immediately after birth; policies that discourage unwrapping and exploring the infant; intensive care environment; restrictive visiting policies; staff indifference or lack of support for the parent's. Allowing the infant and mother to room together, allowing visitors, and working with cultural differences will enable the attachment process to occur.

The nurse is conducting the initial postpartum assessment on a client. The nurse will assist the client into which position to properly assess the postpartum uterus? 1. supine 2. high Fowler 3. left-lateral side lying 4. semi-Fowler

1. supine The best position for a complete assessment of the uterus is lying flat, supine. The other positions will not allow for a true assessment of the location of the uterus in relation to the umbilicus.

A nurse is providing care to a postpartum woman who gave birth about 2 days ago. The client asks the nurse, "I haven't moved my bowels yet. Is this a problem?" Which response by the nurse would be most appropriate? 1. "Let me call your health care provider about this problem." 2. "It might take up to a week for your bowels to return to their normal pattern." 3. "I'll get a laxative prescribed so that you can move your bowels." 4. "That's unusual. Are you making sure to eat enough?"

2. "It might take up to a week for your bowels to return to their normal pattern."

A client diagnosed with pelvic organ prolapse is being taught how to perform pelvic floor muscle exercises. During the teaching session, the client asks the nurse, "How do these exercises help?" Which response by the nurse would be most appropriate? 1. "The exercises help you to establish regular bowel elimination patterns so you don't strain so much." 2. "They help to increase the volume of your muscles, which leads to stronger muscle contraction." 3. "They help to move the pelvic floor upward so that your symptoms eventually decrease." 4. "The exercises increase the amount of blood that your muscles receive, making them less relaxed."

2. "They help to increase the volume of your muscles, which leads to stronger muscle contraction." The purpose of pelvic floor exercises is to increase the muscle volume, which will result in a stronger muscular contraction. The exercises do not move the pelvic floor upward, increase blood supply, or establish regular elimination patterns.

A newborn's vital signs are documented by the nurse and are as follows: HR 144, RR- 36, BP- 128/78, and T- 98.6℉ (37℃). Which finding would be concerning to the nurse? 1. Respiratory Rate 2. Blood Pressure 3. Heart Rate 4. Temperature

2. Blood Pressure The blood pressure of a newborn should be quite low—around 60-70 over 35 to 50. The heart rate and respiratory rate are both high, which are normal findings. The temperature falls within a normal range of 97.7℉ to 99.5℉ (36.5℃ to 37.5℃).

A nurse is called into the room of one of the clients where the grandparents are visiting. The grandmother is visibly upset, and says "Just look at my grandson! His head is all soft and swollen here and it shouldn't be. The doctor injured him when he was born." The nurse assesses the newborn and finds an area of swelling about the size of a half-dollar at the center of the upper scalp. The nurse determines this finding is most likely which condition? 1. Increased intracranial pressure 2. Caput succedaneum 3. Molding 4. Harlequin sign

2. Caput succedaneum Caput succedaneum is swelling of the soft tissue of the scalp caused by pressure of the presenting part on a partially dilated cervix or trauma from a vacuum-assisted delivery. This finding is often of concern for the families. Reassure them that the caput will decrease in a few days without treatment. Increased intracranial pressure would involve the entire scalp and not just a small portion. There would also be other neurologic signs accompanying it. Molding is an elongated head shape caused by overlapping of the cranial bones as the fetus moves through the birth canal. This will also resolve in a few days without treatment. The Harlequin sign is characterized by a clown-suit-like appearance of the newborn where the skin is dark red on one side of the body and the other side is pale. This is a harmless condition which occurs most frequently with vigorous crying or with the infant lying on his or her side.

The nurse is conducting a preadmission class for a group of parents on the safety features that are utilized to help prevent infant abduction. The nurse should prioritize which factor as most essential to ensure the program's success? 1. Use of pass codes onto the unit 2. Cooperation by the parents with the hospital policies 3. Use of monitor attached to babies 4. Use of cameras at all doors

2. Cooperation by the parents with the hospital policies

A newborn infant at 36 hours of age is jaundiced. The mother is breastfeeding. What intervention is appropriate to increase the excretion of bilirubin? 1. Stop breastfeeding and administer glucose water for 24 hours. 2. Instruct the mom to feed every two to three hours. 3. Keep the skin protected by preventing light onto the baby's skin. 4. Restrict feedings and give glucose water every 4-6 hours for hydration.

2. Instruct the mom to feed every two to three hours. Bilirubin is excreted in the urine and feces. Encouraging the mother to breastfeed at least every two to three hours will increase the waste and help decrease the bilirubin level. Stopping breastfeeding and administering glucose water for 24 hours would not be appropriate for the mother. Restricting feedings and giving glucose water every 4 to 6 hours is not an appropriate nursing intervention for an infant showing signs of jaundice. Keeping light away from the baby's skin does not help to clear jaundice; it could only make it worse.

What medication would the nurse administer to a client experiencing uterine atony and bleeding leading to postpartum hemorrhage? 1. Domperidone 2. Oxytocin 3. Calcium gluconate 4. Magnesium sulfate

2. Oxytocin Oxytocin causes the uterus to contract to improve uterine tone and reduce bleeding. Magnesium sulfate is administered to clients with preeclampsia or eclampsia or hypertension problems. Domperidone is used to increase lactation in women. Calcium gluconate is an antagonist used in clients experiencing side effects of magnesium sulfate.

The nurse is documenting assessment of infant reflexes. She strokes the side of the infant's face, and the baby turns toward the stroke. What reflex has the nurse elicited? 1. Moro 2. rooting 3. tonic neck 4. sucking

2. Rooting This is the rooting reflex and is used to encourage the infant to feed. This reflex and the sucking reflex work together to assist the infant with cues for feeding at the breast. The tonic neck (or fencing) reflex and the Moro (or startle reflex) are total body reflexes and assess neurologic function in the newborn.

A postpartum woman has been unable to urinate since giving birth. When the nurse is assessing the woman, which finding would indicate that this client is experiencing bladder distention? 1. Lochia is less than usual. 2. Uterus is boggy. 3. Percussion reveals tympany. 4. Bladder is nonpalpable.

2. Uterus is boggy. A distended bladder is dull on percussion and can be palpated as a rounded mass. In addition, the uterus would be boggy and lochia would be more than usual.

A new mother tells the nurse at the baby's 3 month check-up, "When she cries, it seems like I am the only one who can calm her down." This is an example of which behavior? 1. bonding 2. attachment 3. being spoiled 4. none of the above

2. attachment Attachment is the development of strong affection between an infant and a significant other. It does not occur overnight. It occurs through mutually satisfying experiences. Attachment behaviors include seeking, staying close to, and exchanging gratifying experiences with the infant. Bonding is the close emotional attraction to a newborn by the parents that develops in the first 30 to 60 minutes after birth. This is not an example of being spoiled.

A nurse is reviewing the history of a postpartum woman. The nurse determines that the woman is at low risk for uterine subinvolution based on which findings? Select all that apply. 1. prolonged labor 2. breastfeeding 3. early ambulation 4. hydramnios 5. uterine infection

2. breastfeeding 3. early ambulation Factors that inhibit involution that would result in subinvolution include prolonged labor and difficult birth, uterine infection, overdistention of the uterine muscles such as from hydramnios, a full bladder, close childbirth spacing, and incomplete expulsion of amniotic membranes and placenta. Breastfeeding, early ambulation, and an empty bladder would facilitate uterine involution.

While observing care being provided to an infant, the new mother looks at the nurse repeatedly and asks, "Am I doing this the right way?" Which nursing diagnosis should the nurse select to guide the care needs of the mother at this time? 1. ineffective coping related to expectation to provide newborn care 2. health-seeking behaviors related to care of newborn 3. risk for altered family coping related to an additional family member 4. risk for impaired parenting related to disappointment in the sex of the child

2. health-seeking behaviors related to care of newborn The new mother is asking the nurse to validate actions being performed while providing newborn care. The nursing diagnosis most appropriate for the new mother at this time would be health-seeking behaviors related to care of the newborn. The new mother is not demonstrating signs of ineffective coping. There is no information to support a risk for altered family coping or risk for impaired parenting.

After teaching a group of nurses during an in-service program about risk factors associated with postpartum hemorrhage, the nurse determines that the teaching was successful when the group identifies which risk factor(s)? Select all that apply. 1.use of indwelling catheter 2. hydramnios 3. labor augmentation 4. nulliparity 5. placenta previa

2. hydramnios 3. labor augmentation 5. placenta previa Risk factors for postpartum hemorrhage include precipitous labor less than 3 hours; placenta previa or placental abruption (abruptio placentae); multiparity; uterine overdistention (such as with a large infant, twins, or hydramnios); and labor induction or augmentation. Use of an indwelling catheter is a potential risk for postpartum infection; it is not known to cause risk for postpartum hemorrhage.

A nurse is assessing a client during the postpartum period. Which findings indicate normal postpartum adjustment? Select all that apply. 1. abdominal pain 2. nondistended abdomen 3. active bowel sounds 4. tender abdomen 5. passing gas

2. nondistended abdomen 3. active bowel sounds 5. passing gas Finding active bowel sounds, verification of passing gas, and a nondistended abdomen are normal assessment results. The abdomen should be nontender and soft. Abdominal pain is not a normal assessment finding and should be immediately looked into.

The nurse is observing a client who gave birth yesterday. Where should the nurse expect to find the top of the client's fundus? 1. below the symphysis pubis 2. one fingerbreadth below the umbilicus 3. at the level of the umbilicus 4. one fingerbreadth above the umbilicus

2. one fingerbreadth below the umbilicus After a client gives birth, the height of her fundus should decrease by approximately one fingerbreadth (1 cm) each day. By the end of the first postpartum day, the fundus should be one fingerbreadth below the umbilicus. Immediately after birth, the fundus may be above the umbilicus; 6 to 12 hours after birth, it should be at the level of the umbilicus; 10 days after birth, it should be below the symphysis pubis.

Which finding(s) leads the nurse to suspect that a postpartum woman has developed endometritis? Select all that apply. 1. leukocytosis 2. pain on both sides of the abdomen 3. flank pain 4. foul smelling lochia 5. hematuria

2. pain on both sides of the abdomen 4. foul smelling lochia Signs and symptoms of endometritis can include lower abdominal tenderness or pain on one or both sides, foul-smelling lochia, elevated temperature, accompanying chills, loss of appetite, and general malaise. Leukocytosis may already be present and related to the birthing process, so it is not of significant value in the puerperium. Hematuria and flank pain would be associated with a urinary tract infection.

The nurse is used to working on the postpartum floor taking care of women who have had normal vaginal births. Today, however, the nurse has been assigned to help care for women who are less than 24 hours post-cesarean birth. The nurse realizes that some areas will not be assessed. What would the nurse leave out of the client assessments? 1. lower extremities 2. perineum 3. breasts 4. respiratory status

2. perineum Usually a woman who experiences cesarean birth does not have an episiotomy, although rarely this may be the case.

A nurse is assigned to care for a client with a uterine prolapse. Which of the following would be most important for the nurse to assess when determining the severity of the prolapse? 1. pain in the lower abdomen 2. uterine protrusion into the vagina 3. uterine bleeding present 4. foul-smelling lochia

2. uterine protrusion into the vagina To determine if the uterine prolapse in the client is mild or severe, the nurse should assess for uterine protrusion of the cervix and uterus into the vagina. As more of the uterus descends, the vagina becomes inverted. Uterine bleeding, foul-smelling lochia, and pain or tenderness in the lower abdomen are all characteristic manifestations of late postpartum hemorrhage.

The nurse observes the stool of a newborn who has begun to breastfeed. Which finding would the nurse expect? greenish black, tarry stool yellowish-brown, seedy stool yellowish-green, pasty stool yellow-gold, stringy stool

2. yellowish-brown, seedy stool

The postpartum client and her husband are excited about their new baby. However, they are also concerned about getting pregnant again too soon and ask about using birth control. Which instruction should the nurse include in their discharge education to address this issue? 1. "Ovulation does not return for 6 months after birth." 2. "You will not ovulate until your menstrual cycle returns." 3. "Ovulation may return as soon as 3 weeks after birth." 4. "You may have intercourse until next month with no fear of pregnancy."

3. "Ovulation may return as soon as 3 weeks after birth." Ovulation may start at soon as 3 weeks after birth. The client needs to be aware and use a form of birth control. She needs to be cleared by her health care provider prior to intercourse if she has a vaginal birth, but in the event that she has intercourse, needs to be prepared for the possibility of pregnancy. Ovulation can occur without the return of the menstrual cycle, and ovulation does return sooner than 6 months after birth.

A woman comes to the clinic. She gave birth about 2 months ago to a healthy term male newborn. During the visit, the woman tells the nurse, "I've noticed that I'm a bit uncomfortable now when we have sexual intercourse. Is there anything that I can do?" The woman's menstrual period has not yet resumed. Which suggestion by the nurse would be most appropriate? 1. "This is entirely normal, and many women go through it. It just takes time." 2. "Try doing Kegel exercises to get your pelvic muscles back in shape." 3. "You might try using a water-soluble lubricant to ease the discomfort." 4. "It takes a while to get your body back to its normal function after having a baby."

3. "You might try using a water-soluble lubricant to ease the discomfort." Discomfort during sex and localized dryness usually plague most postpartum women until menstruation returns. Water-soluble lubricants can reduce discomfort during intercourse. Although it may take some time for the woman's body to return to its prepregnant state, telling the woman this does not address her concern. Telling her that dyspareunia is normal and that it takes time to resolve also ignores her concern. Kegel exercises are helpful for improving pelvic floor tone but would have no effect on vaginal dryness.

A breastfeeding mother wants to know how to help her 2-week-old newborn gain the weight lost after birth. Which action should the nurse suggest as the best method to accomplish this goal? 1. Recommend that the mother pump her breast milk and measure it before feeding. 2. Add cereal to the newborn's feedings twice a day. 3. Breastfeed the infant every 2 to 4 hours on demand. 4. Weigh the infant daily to ensure that she is gaining 1.5 to 2 ounces (42.5 to 57 grams) per day.

3. Breastfeed the infant every 2 to 4 hours on demand.

The nurse reviews the history of a postpartum woman G3P3 and notes it is positive for obesity and smoking. The nurse would be especially alert for the development of signs and symptoms of which complication in this client? 1. metritis 2. postpartum hemorrhage 3. deep venous thrombosis 4. uterine atony

3. DVT Factors that can increase a woman's risk for DVT include prolonged bed rest, diabetes, obesity, cesarean birth, progesterone-induced distensibility of the veins of the lower legs during pregnancy, severe anemia, varicose veins, advanced maternal age (older than 34), and multiparity.

The nurse is assisting new parents adjust to the birth of their first child. The parents appear hesitant to pick up the baby, stating they are afraid they will make the baby cry. What is the best response if the nurse discovers the infant is lying relatively still with eyes wide open, looking at the parents? 1. Encourage the mother to breastfeed. 2. Commend the parents for making the right choice. 3. Encourage the parents to pick up the baby. 4. Suggest they rock the baby to sleep.

3. Encourage the parents to pick up the baby. Dr. T. Berry Brazelton's Neonatal Behavioral Scale is often used to note the state of reactivity in newborns. This infant is in the quiet alert state with the eyes open and attentive to people. There is movement, but limited. This is a good time for the parents to interact with the infant, such as picking up the infant, touching, talking, and bonding with the infant. Other states of reactivity include: Active alert: eyes are open and active body movements are present, newborn responds to stimuli with activity Deep sleep: quiet, nonrestless sleep state, newborn is hard to awaken Light sleep: eyes are closed but more activity is noted, newborn moves actively and may show sucking behavior Drowsy: eyes open and close and the eyelids look heavy, body activity is present with intermittent periods of fussiness Crying: eyes may be tightly closed, thrashing movements are made in conjunction with active crying. This would not be the time for the parents to avoid interacting with the infant. There is also no indication the infant is hungry or tired, so feeding or trying to get the infant to go to sleep would also be inappropriate at this time.

The nurse is preparing to administer the ordered injections to a newborn. After noting the mother tested positive for HbsAG, which nursing intervention should the nurse prioritize for the infant? Two doses of the hepatitis B immunoglobulin within 24 hours of birth Hepatitis B vaccination and 2 doses of hepatitis B immunoglobulin within 24 hours of birth 3. Hepatitis B vaccination and 1 dose of hepatitis B immunoglobulin within 12 hours of birth Hepatitis B vaccination and 1 dose of hepatitis B immunoglobulin within 24 hours of birth

3. Hepatitis B vaccination and 1 dose of hepatitis B immunoglobulin within 12 hours of birth If a mother has hepatitis B (HbsAG) or is suspected of having hepatitis B, the newborn should be bathed and then should receive 1 dose of the hepatitis B vaccine and 1 dose of the hepatitis B immunoglobulin within 12 hours of birth. The other choices are the wrong dosages and/or times.

A client who has a breastfeeding newborn reports sore nipples. Which intervention can the nurse suggest to alleviate the client's condition? 1. Recommend a moisturizing soap to clean the nipples. 2. Fasten nursing bra flaps immediately after feeding. 3. Offer suggestions based on observation to correct positioning or latching. 4. Encourage use of breast pads with plastic liners.

3. Offer suggestions based on observation to correct positioning or latching. The nurse should observe positioning and latching-on technique while breastfeeding so that she may offer suggestions based on observation to correct positioning/latching. This will help minimize trauma to the breast. The client should use only water, not soap, to clean the nipples to prevent dryness. Breast pads with plastic liners should be avoided. Leaving the nursing bra flaps down after feeding allows nipples to air dry.

The nurse is preparing a nursing care plan for an immediate postpartum client. Which nursing diagnosis should the nurse prioritize? 1. Acute pain related to afterpains or episiotomy discomfort 2. Risk for infection related to multiple portals of entry for pathogens 3. Risk for injury: postpartum hemorrhage related to uterine atony 4. Risk for injury: falls related to postural hypotension and fainting

3. Risk for injury: postpartum hemorrhage related to uterine atony The highest priority is the risk for injury related to postpartum hemorrhage. The client needs close observation and assessment for hemorrhage. All of the options presented are appropriate nursing diagnoses for a postpartum client. However, the other options do not take precedence over the risk for postpartum hemorrhage.

A nurse helps a postpartum woman out of bed for the first time postpartum and notices that she has a very heavy lochia flow. Which assessment finding would best help the nurse decide that the flow is within normal limits? 1. The flow contains large clots. 2. Her uterus is soft to your touch. 3. The color of the flow is red. 4. The flow is over 500 mL.

3. The color of the flow is red. A typical lochia flow on the first day postpartally is red; it contains no large clots; the uterus is firm, indicating that it is well contracted.

When assessing the postpartum client 2 hours after giving birth, which finding indicates the need for further action? 1. The fundus is firm and located one fingerbreadth below the level of the umbilicus. 2. The fundus is firm and located at the level of the umbilicus. 3. The fundus is firm and deviated sharply to the right side of the abdomen. 4. The fundus is firm and located 1 fingerbreadth above the level of the umbilicus.

3. The fundus is firm and deviated sharply to the right side of the abdomen. In the immediate postpartum period, the fundus is regularly assessed. The fundus must be firm. A boggy fundus indicates uterine atony and will result in blood loss. The fundus is to be midline in the abdomen. A deviation to the side may indicate a full bladder. In the immediate hours after birth, the fundus may be found at one fingerbreadth above or below the umbilicus.

Following delivery, a newborn has a large amount of mucus coming out of his mouth and nose. What would be the nurse's first action? Suction the mouth and then the nose with a suction catheter. Suction the nose first and then the mouth with a bulb syringe. Using a bulb syringe, suction the mouth then the nose. Place the newborn on its stomach with the head down and gently pat its back.

3. Using a bulb syringe, suction the mouth then the nose. A bulb syringe is used initially to suction secretions from a newborn's mouth and nose, starting with the mouth so the newborn does not aspirate the mucus into its lungs. Suctioning the nose first may stimulate the newborn to gasp or cry and this may lead to aspiration. A suction catheter is only used if the bulb syringe cannot manage all the secretions. Patting the newborn on the back will not clear out all the oral secretions.

A nurse is caring for a non-breastfeeding client in the postpartum period. The client reports engorgement. What suggestion should the nurse provide to alleviate breast discomfort? 1. Apply warm compresses. 2. Express milk frequently. 3. Wear a well-fitting bra. 4. Apply hydrogel dressing.

3. Wear a well-fitting bra. The nurse should suggest the client wear a well-fitting bra to provide support and help alleviate breast discomfort. Application of warm compresses and expressing milk frequently are suggested to alleviate breast engorgement in breastfeeding clients. Hydrogel dressings are used prophylactically in treating nipple pain.

The night shift LPN is checking on a woman who had a cesarean birth with spinal morphine injection anesthesia early that morning. The nurse counts a respiratory rate of 8 per minute. What should the nurse do first? 1. perform bag-to-mouth rescue breathing at a rate of 12 per minute 2. awaken the woman and instruct her to breathe more rapidly 3. administer naloxone per the preprinted orders 4. call the anesthesiologist from the room for orders

3. administer naloxone per the preprinted orders Have naloxone readily available. The anesthesiologist orders naloxone administration if the respiratory rate falls below 10 to 12 per minute.

A nurse finds the uterus of a postpartum woman to be boggy and somewhat relaxed. This a sign of which condition? 1. normal involution 2. hemorrhage 3. atony 4. infection

3. atony The uterus in a postpartum client should be midline and firm. A boggy or relaxed uterus signifies uterine atony, which can predispose the woman to hemorrhage.

When the nurse is assessing a postpartum client approximately 6 hours after birth, which finding would warrant further investigation? 1. voiding of 350 cc 2. deep red, fleshy-smelling lochia 3. blood pressure 90/50 mm Hg 4. profuse sweating

3. blood pressure 90/50 mm Hg In most instances of postpartum hemorrhage, blood pressure and cardiac output remain increased because of the compensatory increase in heart rate. Thus, a decrease in blood pressure and cardiac output are not expected changes during the postpartum period. Early identification is essential to ensure prompt intervention. Deep red, fleshy-smelling lochia is a normal finding 6 hours postpartum. Voiding in small amounts such as less than 150 cc would indicate a problem, but 350 cc would be appropriate. Profuse sweating also is normal during the postpartum period.

During a routine assessment the nurse notes the postpartum client is tachycardic. What is a possible cause of tachycardia? 1. bladder distention 2. uterine atony 3. delayed hemorrhage 4. extreme diaphoresis

3. delayed hemorrhage Tachycardia in the postpartum woman can suggest anxiety, excitement, fatigue, pain, excessive blood loss or delayed hemorrhage, infection, or underlying cardiac problems. Further investigation is always warranted to rule out complications. An inability to void would suggest bladder distention. Extreme diaphoresis would be expected as the body rids itself of excess fluid. Uterine atony would be associated with a boggy uterus and excess lochia flow.

Assessment of a postpartum client reveals a firm uterus with bright-red bleeding and a localized bluish bulging area just under the skin at the perineum. The woman also reports significant pelvic pain and is experiencing problems with voiding. The nurse suspects which condition? 1. bladder distention 2. laceration 3. hematoma 4. uterine atony

3. hematoma The woman most likely has a hematoma based on the findings: firm uterus with bright-red bleeding; localized bluish bulging area just under the skin surface in the perineal area; severe perineal or pelvic pain; and difficulty voiding. A laceration would involve a firm uterus with a steady stream or trickle of unclotted bright-red blood in the perineum. Bladder distention would be palpable along with a soft, boggy uterus that deviates from the midline. Uterine atony would be noted by a uncontracted uterus.

The nurse is assessing a postpartum woman and is concerned the client may be hemorrhaging. Which assessment finding is the nurse finding most concerning? 1. increased blood pressure 2. increased hematocrit level 3. increased heart rate 4. increased cardiac output

3. increased heart rate Tachycardia in the postpartum woman warrants further investigation as it can indicate postpartum hemorrhage. Typically the postpartum woman is bradycardic for the first 2 weeks. In most instances of postpartum hemorrhage, blood pressure and cardiac output remain increased because of a compensatory increase in heart rate. Hypotension would be another concerning assessment, especially orthostatic hypotension, as it can also indicate hemorrhage. Red blood cell production ceases early in the postpartum period, causing hemoglobin and hematocrit levels to decrease slightly in the first 24 hours and then rise slowly. Hematocrit would be unreliable as an indicator of hemorrhage.

A nurse is providing discharge instructions to a postpartum client about possible complications after returning home. For which finding will the client contact the health care provider? 1. decreasing amount and lightening of the color of lochia 2. passing clots smaller than 1 inch (2.5 cm) 3. increasing amount and darkening of the color of lochia 4. palpating a firm fundus

3. increasing amount and darkening of the color of lochia Once the lochia has changed to pink, a change back to a darker color may indicate a complication. The client does not need to contact the health care provider for normal findings, such as a firm fundus, passing clots smaller than 1 in (2.5 cm), and decreasing amount of lochia that is lighter in color.

The nurse assesses a postpartum client's discharge as being moderate in amount and red in color. How should the nurse document the appearance of the lochia? 1. lochia normalia 2. lochia serosa 3. lochia rubra 4. lochia alba

3. lochia rubra Lochia that is red in color, or bloody, is termed lochia rubra. Lochia alba is colorless flow that occurs around postpartum day 10. Lochia serosa is pink or brown in color and appears around postpartum day 4. Lochia normalia is not a term used to describe lochia.

A primipara client gave birth vaginally to a healthy newborn girl 48 hours ago. The nurse palpates the client's fundus and documents which finding as normal? 1. two fingerbreadths above the umbilicus 2. at the level of the umbilicus 3. two fingerbreadths below the umbilicus 4. four fingerbreadths below the umbilicus

3. two fingerbreadths below the umbilicus During the first few days after birth, the uterus typically descends downward from the level of the umbilicus at a rate of 1 cm (1 fingerbreadth) per day so that by day 2, it is about 2 fingerbreadths below the umbilicus.

Which factor in a client's history would alert the nurse to an increased risk for postpartum hemorrhage? 1. Multiparity, age of mother, operative delivery 2. Size of placenta, small baby, operative delivery 3. Uterine atony, placenta previa, operative procedures 4. Prematurity, infection, length of labor

3. uterine atony, placenta previa, operative procedures Risk factors for postpartum hemorrhage include a precipitous labor less than three hours, uterine atony, placenta previa or abruption, labor induction or augmentation, operative procedures such as vacuum extraction, forceps, or cesarean birth, retained placental fragments, prolonged third stage of labor greater than 30 minutes, multiparity, and uterine overdistention such as from a large infant, twins, or hydramnios.

The nurse is concerned that a postpartum client with a cervical laceration is developing hypovolemic shock. What did the nurse assess in this client? 1. decreased respiratory rate 2. elevated blood pressure 3. weak and rapid pulse 4. warm and flushed skin

3. weak and rapid pulse If the loss of blood is extremely copious, a woman will quickly begin to exhibit symptoms of hypovolemic shock such as a weak and rapid pulse. The skin will be pale and clammy, and the blood pressure will fall. Respiratory rate will be increased and shallow.

A client who recently gave birth to her third child expresses a desire to have her older two children come to the hospital for a visit. What should the nurse say in response to this request? 1. "I recommend that you introduce the new baby to her siblings once you are back at home. Right now you need to rest and recover." 2. "Your baby is so vulnerable to infections right now that it would be better to wait until you are at home to introduce her to her siblings." 3. "That's a great idea! They can also take the baby out into the hall and walk with it for a while to give you a break." 4. "As long as they are well, absolutely. Why don't we give you a dose of pain medication beforehand so that you will enjoy the visit?"

4. "As long as they are well, absolutely. Why don't we give you a dose of pain medication beforehand so that you will enjoy the visit?" Separation from children is often as painful for a mother as it is for her children. A chance to visit the hospital and see the new baby and their mother reduces feelings that their mother cares more about the new baby than about them. It can help to not only relieve some of the impact of separation but also to make the baby a part of the family. Assess to be certain siblings are free of contagious diseases such as upper respiratory tract illnesses or recent exposure to chickenpox before they visit. Then, have them wash their hands and, if they choose, hold or touch the newborn with parental assistance. Allowing the siblings to walk with the baby out in the hall unsupervised would be unsafe.

A nurse teaches new parents how to soothe a crying newborn. Which statement by the parents indicates to the nurse the teaching was effective? 1. "We will vigorously rub our baby's back as we play some music." 2. "We will place our baby on the belly on a blanket on the floor." 3. "We will hold feedings until our baby stops crying." 4. "We will turn the mobile on that's hanging on our baby's crib."

4. "We will turn the mobile on that's hanging on our baby's crib." Turning on a mobile above the newborn's head is helpful in calming the newborn. The movement is distracting, and the music is comforting. The newborn's back should be rubbed lightly while the parents speak softly or play calming music or white noise. Swaddling the newborn rather than placing the infant on a blanket on the floor provides security and comfort. Feeding or burping can be helpful in relieving air or stomach gas.

A client has been discharged from the hospital after a cesarean birth. Which instruction should the nurse include in the discharge teaching? 1. "Call your health care provider if you saturate a peri-pad in less than 4 hours." 2. "Follow up with your health care provider within 3 weeks of being discharged." 3. Notify the health care provider if your temperature is greater than 99° F (37.2° C)." 4. "You should be seen by your health care provider if you have blurred vision."

4. "You should be seen by your health care provider if you have blurred vision." The client needs to notify the health care provider for blurred vision, as this can indicate preeclampsia in the postpartum period. The client should also notify the health care provider if she has a temperature great than 100.4° F (38° C) or if a peri-pad is saturated in less than 1 hour. The nurse should ensure that the follow-up appointment is within 2 weeks after hospital discharge.

The nurse is caring for a newborn who was delivered via a planned cesarean birth. The nurse determines the infant requires closer monitoring than a vaginal delivery infant based on which factor? 1. Much of the fetal lung fluid is squeezed out in cesarean birth. 2. Fetal lungs are uninflated and full of amniotic fluid that must be absorbed. 3. Oxygen was cut off when the umbilical cord was clamped, resulting in decreased oxygen and increased carbon dioxide. 4. Excessive fluid in the infant's lungs, making respiratory adaptation more challenging

4. Excessive fluid in the infant's lungs, making respiratory adaptation more challenging During a vaginal birth the infant is squeezed by uterine contractions, which squeeze fluid out of the lungs and prepare them for breathing. The infant who is born via cesarean without labor first does not have the mechanical removal of the fluid from the lungs. This places the infant at increased risk for respiratory compromise, so there is a need to more closely assess a newborn after birth. The lungs should inflate once the baby is delivered and not wait until the amniotic fluid is absorbed. The umbilical cord is not clamped until the infant is out of the womb and starts to take its first breaths.

A postpartum woman is prescribed oxytocin to stimulate the uterus to contract. Which action would be most important for the nurse to do? 1. Withhold the drug if the woman is hypertensive. 2. Administer the drug as an IV bolus injection. 3. Give as a vaginal or rectal suppository. 4. Piggyback the IV infusion into a primary line.

4. Piggyback the IV infusion into a primary line. When giving oxytocin, it should be diluted in a liter of IV solution and the infusion set up to be piggy-backed into a primary line to ensure that the medication can be discontinued readily if hyperstimulation or adverse effects occur. It should never be given as an IV bolus injection. Oxytocin may be given if the woman is hypertensive. Oxytocin is not available as a vaginal or rectal suppository.

Which action will the nurse avoid when performing basic care for a newborn male? 1. Determining the location of the urethral opening 2. Palpating if testes are descended into the scrotal sac 3. Inspecting the genital area for irritated skin 4. Retracting the foreskin over the glans to assess for secretions

4. Retracting the foreskin over the glans to assess for secretions The foreskin in male newborns does not normally retract and should not be forced. The nurse will inspect the genital area for irritated skin to prevent and/or treat possible skin irritations. The nurse will palpate the testes to determine if the newborn has cryptorchidism. It is important to verify that the urethral opening is at the tip of the glans and not on the dorsal or ventral sides as these would need intervention. This can be accomplished without overmanipulating the foreskin.

What physiologic changes occur after birth when the cord is cut and clamped? 1. The oxygenated blood coming from the placenta is diverted around the liver through the ductus venosus. 2. The placenta is immediately expelled. 3. The ductus ovale closes over the next 2 to 3 days. 4. The infant takes its first breath and the lungs expand to increase blood oxygen levels.

4. The infant takes its first breath and the lungs expand to increase blood oxygen levels. As the infant takes its first breath, the pulmonary resistance decreases and blood flow increases to the lungs, resulting in increased oxygenation of the infant's blood. The ductus venosus and the ductus arteriosus close, not the ductus ovale. The foramen ovale normally closes when the pressure gradient in the heart shift from higher pressure on the right side to higher pressure on the left side of the heart. The placenta is not immediately expelled once the infant takes its first breath.

In recording a postpartum mother's urinary output, the nurse notes that she is voiding between 150 and 200 ml with each hourly void. How would the nurse interpret this finding? 1. The urinary output is above expected levels. 2. The urinary output is inadequate and the mother needs to drinks more fluids. 3. The urinary output is inadequate suggestive of urinary retention. 4. The urinary output is normal.

4. The urinary output is normal

A first-time mother informs the nurse that she is unable to breastfeed her newborn through the day as she is usually away at work. She adds that she wants to express her breast milk and store it for her newborn to have later. What instruction would be correct to offer the mother to ensure the safety of the stored expressed breast milk? 1. Refreeze any unused milk for later use if it has not been out more that 2 hours. 2. Use microwave ovens to warm the chilled milk. 3. Use any frozen milk within 6 months of obtaining it. 4. Use the sealed and chilled milk within 24 hours.

4. Use the sealed and chilled milk within 24 hours. The nurse should instruct the woman to use the sealed and chilled milk within 24 hours. The nurse should not instruct the woman to use frozen milk within 6 months of obtaining it, to use microwave ovens to warm chilled milk, or to refreeze the used milk and reuse it. Instead, the nurse should instruct the woman to use frozen milk within 3 months of obtaining it, to avoid using microwave ovens to warm chilled milk, and to discard any used milk and never refreeze it.

The nurse explains to the parents of a 2-day-old newborn that decreased life span of neonatal red blood cells has contributed to which complication? transient tachypnea of the newborn polycythemia respiratory distress syndrome hyperbilirubinemia

4. hyperbilirubinemia Neonatal red blood cells have a life span of 80 to 100 days and normally have a higher count at birth. This combination leads to increased hemolysis. Complications of this process include hyperbilirubinemia.

A woman who is breastfeeding her newborn reports that her breasts seem quite full. Assessment reveals that her breasts are engorged. Which factor would the nurse identify as the most likely cause for this development? 1. cracking of the nipple 2. improper positioning of infant 3. inadequate secretion of prolactin 4. inability of infant to empty breasts

4. inability of infant to empty breasts For the breastfeeding mother, engorgement is often the result of vascular congestion and milk stasis, primarily caused by the infant not fully emptying the mother's breasts at each feeding. Cracking of the nipple could lead to infection. Improper positioning may lead to nipple tenderness or pain. Inadequate secretion of prolactin causes a decrease in the production of milk.

When caring for a mother who has had a cesarean birth, the nurse would expect the client's lochia to be: 1. saturated with clots and mucus. 2. about the same as after a vaginal birth. 3. greater than after a vaginal birth. 4. less than after a vaginal birth.

4. less than after a vaginal birth. Women who have had cesarean births tend to have less flow because the uterine debris is removed manually along with delivery of the placenta.

A nurse is teaching newborn care to students. The nurse correctly identifies which mechanism as the predominant form of heat loss in the newborn? 1. nonshivering thermogenesis 2. lack of brown adipose tissue 3. sweating and peripheral vasoconstriction 4. radiation, convection, and conduction

4. radiation, convection, and conduction Heat loss in the newborn occurs primarily through radiation, convection, and conduction because of the newborn's large ratio of body surface to weight and because of the marked difference between core and skin temperatures. Nonshivering thermogenesis is a mechanism of heat production in the newborn. Lack of brown adipose tissue contributes to heat loss, particularly in premature infants, but it is not the predominant form of heat loss. Peripheral vasoconstriction is a method to increase heat production.

A nurse needs to monitor the blood glucose levels of a newborn under observation at a health care facility. When should the nurse check the newborn's initial glucose level? 1. on admission to the nursery 2. 4 hours after admission to the nursery 3. 24 hours after admission to the nursery 4. after the newborn has received the initial feeding

Typically, a newborn's blood glucose levels are assessed with use of a heel stick sample of blood on admission to the nursery, not 4 or 24 hours after admission to the nursery. It is also not necessary or even reasonable to check the glucose level only after the newborn has been fed.


Ensembles d'études connexes

intro to supply chain Taitt chapter 3

View Set

Chapter 11: Stress on Health/Well Being

View Set

Safety and Mobility - GERONTOLOGY

View Set

Maynard Exam #1 Reading Quiz Questions

View Set

Pediatrics EOR ORTHOPEDICS/RHEUMATOLOGY

View Set